Módulo 4 Med. Interna Flashcards

1
Q

A 21-year-old man is brought by his roommate to the emergency department because of abrupt onset of shortness of breath, mild chest pain, and a sensation of rapid heart beating. The patient says that in the past he had similar episodes, which resolved with the Valsalva maneuver or breath holding. This time, these measures were unsuccessful. He does not take any medication and is otherwise in good health. An ECG documents supraventricular tachycardia with a pulse of 200/min. Under ECG monitoring, gentle massage over the right carotid sinus is attempted, but the attack does not cease. Which of the following is the most appropriate next step in treatment?

(A) Further carotid sinus massage
(B) IV lidocaine
(C) IV procainamide
(D) IV verapamil
(E) Oral verapamil

A

Respuesta: D

The correct answer is D. Paroxysmal supraventricular tachycardia is the most common paroxysmal arrhythmia with a rapid heart rate. It is often associated with a perfectly normal heart. Depending on heart rate, manifestations may vary from a subjective sensation of increased heart rate to mild chest pain, shortness of breath, or syncope. The pulse is usually between 160 and 220/min. Patients may be instructed to carry out maneuvers that stimulate the vagal nerve (e.g., Valsalva maneuver, breath holding, and arm and body stretching) and may succeed in interrupting the attacks. Carotid sinus massage should be performed for 10-20 seconds on the patient in a semirecumbent position and only on one side. Presence of carotid bruits is an absolute contraindication to carotid sinus massage. IV verapamil (a calcium channel blocker) or IV adenosine is the treatment of choice once other nonpharmacologic measures have been tried.

Further carotid sinus massage (choice A) would most probably be unsuccessful.

IV lidocaine (choice B) is reserved for treatment of ventricular tachyarrhythmias, especially in the setting of acute myocardial infarction.

IV procainamide (choice C) is not a choice for treatment of supraventricular tachycardia. Oral procainamide may be used for prevention of attacks as an alternative to digoxin, verapamil, or beta blockers.

Oral verapamil, 80-120 mg/4-6 hours (choice E), can be tried on patients with mild symptoms, but this is not the case in this patient.

How well did you know this?
1
Not at all
2
3
4
5
Perfectly
2
Q

A 45-year-old African American man is taken to the emergency department because he is vomiting fresh blood. His temperature is 37 C (98.6 F), blood pressure is 65/30 mm Hg, pulse is 120/min, and respirations are 24/min. The patient is stabilized, then taken for emergency endoscopy. The source of bleeding is a tortuous vein near the gastroesophageal junction. The bleeding is successfully stopped by banding of the vessel. Which of the following is the most likely underlying condition predisposing this patient for this complication?

(A) Alcoholism
(B) Alpha1-antitrypsin deficiency
(C) Hemochromatosis
(D) Hepatitis A infection
(E) Hepatitis B infection

A

Respuesta: A

The correct answer is A. The patient has a bleeding esophageal varix. These are dilated vessels that usually develop when cirrhosis blocks blood flow from the portal venous system through the liver, and back to the systemic circulation. Hypersplenism and arteriovenous malformations that markedly increase blood flow in the portal system can also cause esophageal varices, even in the absence of cirrhosis. Although any type of cirrhosis can potentially cause bleeding esophageal varices, you should be aware that the most common cause is alcoholism, possibly because it is both very common and may be accompanied by gastritis, which makes it easier for the varices to start to bleed. While individual incidents of bleeding varices can often be handled by either sclerotherapy or endoscopic banding of the bleeding vessel, the long-term prognosis remains poor for these patients, who often eventually either exsanguinate or die of other complications of advanced cirrhosis.

Alpha1-antitrypsin deficiency (choice B) is a rare cause of cirrhosis.

Hemochromatosis (choice C) is much less common than alcoholism as a cause of cirrhosis.

Hepatitis A infection (choice D) may cause fulminant hepatic failure but does not cause cirrhosis.

Hepatitis B infection (choice E) is also a relatively common cause of cirrhosis, but it is not as strongly linked to bleeding varices as is alcoholism. In real life, coexistent alcoholism and hepatitis B infection are common.

How well did you know this?
1
Not at all
2
3
4
5
Perfectly
3
Q

A 32-year-old man with AIDS develops right-sided weakness over the course of 1 week. He is on a combined drug regimen of zidovudine (AZT) and a protease inhibitor, and his CD4 cell count is 190 cells/mL. MRI of the brain reveals a single 2-cm mass in the left cerebral white matter that appears as an area of low signal surrounded by a rim of contrast enhancement (“ring-enhancing lesion”). A trial of sulfadiazine and pyrimethamine is started. Three weeks after beginning this treatment, the patient’s neurologic status is unchanged, and imaging studies show that the lesion has not regressed. Which of the following is the most likely diagnosis?

(A) Cryptococcal meningoencephalitis
(B) Glioblastoma multiforme (GBM)
(C) Metastatic tumor
(D) Primary brain lymphoma
(E) Toxoplasma abscess

A

Respuesta: D

The correct answer is D. The two most common causes of intracranial masses in AIDS patients are cerebral toxoplasmosis and primary brain lymphoma, both of which usually manifest as ring- enhancing lesions on CT or MRI. The central nonenhancing area of the lesion consists of necrosis, whereas the peripheral rim is due to viable tissue, inflammatory or neoplastic. A single ring-enhancing lesion in an AIDS patient is treated with a full course of anti- Toxoplasma agents, e.g., sulfadiazine and pyrimethamine. If there is no response following 3 weeks of therapy, an alternative diagnosis of primary brain lymphoma is investigated, and a brain biopsy is performed, if possible, to confirm the clinical diagnosis.

Cryptococcal meningoencephalitis (choice A) is a frequent opportunistic infection associated with AIDS. It manifests with diffuse involvement of the leptomeninges, not with an intracerebral space-occupying lesion. The patient is severely ill and may become rapidly comatose, although localizing signs (meningismus) may be mild.

Glioblastoma multiforme (GBM) (choice B) may also give rise to a nonenhancing lesion on MRI, but its incidence is not increased in AIDS patients compared with immune competent individuals. GBM would be unusual in a young adult.

Metastatic tumor (choice C) is sometimes associated with the same MRI appearance as abscess or GBM. In an AIDS patient, however, metastases occur much less frequently than toxoplasmosis or lymphoma. Furthermore, metastases are usually multiple and located at the gray-white matter junction.

A Toxoplasma abscess (choice E) responds to sulfadiazine and pyrimethamine treatment and shows signs of shrinkage on follow-up MRI.

How well did you know this?
1
Not at all
2
3
4
5
Perfectly
4
Q

A 68-year-old woman complains of 2 days of cough and purulent sputum and right-sided chest pain exacerbated by breathing. She has had middle back pain unassociated with trauma for the past 2 months. One month ago, she was hospitalized briefly for pneumonia. On physical examination, her temperature is 39.0 C (102.2 F), blood pressure is 110/80 mm Hg, pulse is 98/min, and respirations are 28/min. There are crackles and egophony over the right lower lung field. There is no occult blood in the stool. Blood tests show a white blood cell count of 16,000/mm3 , a hematocrit of 18%, and a platelet count of 189,000/mm3 . The mean corpuscular volume is 82 μm3 . A chest x-ray film shows consolidation of the right middle and lower lobes, diffuse osteopenia, and multiple lytic lesions of the ribs and thoracic spine. Which of the following is the most likely laboratory finding?

(A) Decreased serum ferritin
(B) Elevated serum protein
(C) Hyperphosphatemia
(D) Hypertriglyceridemia
(E) Hypocalcemia

A

Respuesta: B

The correct answer is B. This case describes the presentation of multiple myeloma, a plasma cell dyscrasia, characterized by multiple bone marrow tumor foci. Multiple myeloma is associated with osteomalacia, hypercalcemia, and normochromic, normocytic anemia. Susceptibility to infections occurs because of depression of immunoglobulin levels. Pneumonia and pyelonephritis are the most common types of bacterial infection. Elevated serum protein is caused by clonal overproduction of immunoglobulin.

Patients often are anemic because of bone marrow suppression of erythrocyte production. The iron level is not affected. In addition, this patient’s mean corpuscular volume is greater than 80 μm3 , which is not consistent with iron deficiency. Therefore, decreased serum ferritin (choice A) would not be expected.

Hyperphosphatemia (choice C) is not associated with multiple myeloma but is seen with disorders of the parathyroid.

Hypertriglyceridemia (choice D) is not related to multiple myeloma but is a disorder of lipid metabolism. Multiple myeloma is associated with hypercalcemia (due to lytic effects on bone), not hypocalcemia (choice E).

How well did you know this?
1
Not at all
2
3
4
5
Perfectly
5
Q

A 38-year-old woman is complaining of shortness of breath that started suddenly on the morning of presentation. She is an otherwise healthy woman. She takes oral contraceptive pills, and she has a 10- year history of smoking a pack of cigarettes daily. She appears anxious and is rapidly breathing at 30 breaths/min. Her pulse is 110/min, and her blood pressure is 120/80 mm Hg and stable. The rest of her physical examination is unremarkable. Which of the following is the most appropriate initial step in management?

(A) Aspirin
(B) Coumadin
(C) Heparin
(D) IV fluid
(E) Streptokinase

A

Respuesta. C

The correct answer is C. This patient probably has a pulmonary embolism (PE). Oral contraceptive pills and smoking place her at an increased risk for thromboembolic disease. The most common symptoms of PE include tachypnea and tachycardia. Shock may also ensue, and a massive PE can result in loss of blood pressure. This patient has a stable blood pressure at this time. Thus, placing her on heparin immediately would be appropriate to prevent the clot burden from further increasing.

Aspirin (choice A) would succeed in inhibiting platelet aggregation and would be appropriate if the patient had a coronary syndrome, such as myocardial infarction.

Coumadin (choice B) would be appropriate as a long-term choice for anticoagulation in this patient. However, it would take several days before the drug would be an effective anticoagulant and would not be of use in this acute setting. Ultimately, this patient would be taken off heparin and switched to Warfarin.

IV fluids (choice D) would be useful if this patient’s blood pressure were falling in order to support her circulation. Since she is hemodynamically stable, she needs heparin before any other supportive therapy.

Streptokinase (choice E) could be used to immediately lyse the PE, but this would be indicated only if the patient were hemodynamically unstable with a low blood pressure or refractory hypoxemia. Because she is currently maintaining her blood pressure and is not exhibiting signs of right heart failure, streptokinase need not be given, avoiding the associated risk of bleeding and stroke.

How well did you know this?
1
Not at all
2
3
4
5
Perfectly
6
Q

A 35-year-old man is brought to the emergency department after he faints and cannot be revived. Stat chemistries are notable for a plasma glucose of 23 mg/dL. The patient is promptly given IV glucose and recovers consciousness. Careful questioning reveals that he has a family history of endocrine abnormalities. Follow-up studies performed on the remainder of the blood drawn for the screening studies demonstrate insulin levels of 120 mU/mL (reference range 5-25 mU/mL). A CT scan of the abdomen demonstrates a 2-cm mass in the tail of the pancreas. Which additional finding will most likely be seen this patient?

(A) Marfanoid habitus
(B) Medullary carcinoma of the thyroid
(C) Mucosal neuromas
(D) Parathyroid adenoma
(E) Pheochromocytoma

A

Respuesta: D

The correct answer is D. This is a question about multiple endocrine neoplasia (MEN) type I, also known as Wermer syndrome. The patient has a probable pancreatic islet cell tumor that is secreting insulin. In MEN I, 30 to 75% of the patients develop pancreatic islet cell tumors, and about 40% of those develop insulin secreting tumors. The remainder of the islet cell tumors are derived from non-B cell elements and can secrete a variety of substances, most commonly gastrin (producing the multiple peptic ulcers of Zollinger-Ellison syndrome). Other features of MEN 1, which may occur either sequentially or concurrently, include parathyroid adenomas (more than 90% of cases) and pituitary adenomas (50 to 65% of cases). Parathyroid adenomas are also found in 25% of MEN IIA cases and rarely in MEN IIB. Pancreatic islet cell tumors and pituitary adenomas are not a feature of MEN IIA or MEN IIB.

Marfanoid habitus (choice A) is a feature of MEN IIB.

Medullary carcinoma of the thyroid (choice B) is a feature of MEN IIA and MEN IIB.

Mucosal neuromas (choice C) are a feature of MEN IIB.

Pheochromocytoma (choice E) is a feature of MEN IIA and MEN IIB.

How well did you know this?
1
Not at all
2
3
4
5
Perfectly
7
Q

A 45-year-old man presents to a physician because of repeated episodes of fainting. The radial pulse is erratic, with multiple skipped beats at a rate of 45 beats/min. An ECG shows a normal sinus rhythm at a rate of 60/min. An echocardiogram reveals a 2-cm mass that is acting like a “ball valve” to produce intermittent obstruction of flow. Which of the following is the most likely location of this patient’s lesion?

(A) Aorta
(B) Left atrium
(C) Left ventricle
(D) Right atrium
(E) Right ventricle

A

Respuesta: B

The correct answer is B. The most common primary cardiac tumor of adults is benign atrial myxoma. Ninety percent of these lesions involve the left atrium, where they can produce intermittent obstruction when they prolapse into the mitral orifice during diastole. Resection is curative. Other tumors occur less commonly.

Lipomas may involve the left ventricle (choice C), right atrium (choice D), or atrial septum.

Rhabdomyomas are found in children and usually involve the left ventricle (choice C) or the right ventricle (choice E).

The aorta (choice A) is not a common site of tumor formation.

How well did you know this?
1
Not at all
2
3
4
5
Perfectly
8
Q

A diabetic patient is undergoing a routine physical examination. During the review of systems, the patient comments that he has been having pain on swallowing. He localizes the pain to below his sternum. Barium swallow shows slightly raised plaques throughout the esophagus. Endoscopy demonstrates plaques covered with white, curdy, cheese-like material. A biopsy of one of the lesions will most likely demonstrate which of the following?

(A) Acute-angled, branching, septated filaments
(B) Anaplastic squamous cells with numerous mitotic figures
(C) Intranuclear and cytoplasmic inclusion bodies with “owl’s-eye appearance
(D) Loss of ganglion cells in the myenteric plexus
(E) Multinucleated epithelial giant cells on Giemsa stain
(F) Pseudohyphal mycelia and budding yeast cells

A

Respuesta: F

The correct answer is F. White, curdy, cheese-like material specifically suggests thrush due to Candida albicans, whether it occurs in the mouth, vagina, or esophagus. This fungus occurs in both yeast and fungal forms, with both hyphae and pseudohyphae and budding yeast cells. Candida is usually a mouth and vagina commensal organism but can cause clinical disease, particularly among the immunosuppressed. Diabetics are particularly likely to develop clinical Candida infections because they are both immunosuppressed and have body secretions with a high sugar content on which the fungi like to feed.

Acute-angled, branching, septated filaments (choice A) describe Aspergillus, a much less common esophageal pathogen seen most often in AIDS patients. Although it can produce mucosal thickening, it does not cause the white, cheesy curd of Candida.

Anaplastic squamous cells with numerous mitotic figures (choice B) describe squamous cell carcinoma of the esophagus. Diabetics are at no greater risk for esophageal carcinoma. Risk factors include alcohol use and cigarette smoking. Esophageal carcinoma typically presents with weight loss and progressive difficulty in swallowing.

Intranuclear and cytoplasmic inclusion bodies with “owl’s-eye” appearance (choice C) describe cytomegalovirus, a cause of esophageal ulcers, especially in AIDS patients.

Achalasia presents with pain upon swallowing, and a loss of ganglion cells in the myenteric plexus (choice D) is seen on biopsy. However, there are are no visible lesions seen on endoscopy, and there is no association with diabetes.

Multinucleated epithelial giant cells on Giemsa stain (choice E) describe herpes simplex virus, a cause of esophageal ulcers in AIDS patients.

How well did you know this?
1
Not at all
2
3
4
5
Perfectly
9
Q

A 45-year-old woman comes to the physician because of persistent blurred vision for the past month. She also reports three episodes of Candida vaginitis during the past year. She is 167 cm (66 in) tall, and weighs 84 kg (185 lb). Her blood pressure is 130/84 mm Hg. Funduscopic examination reveals dot retinal hemorrhages and increased tortuosity of retinal veins. Her family history is significant for obesity, coronary artery disease, and type 2 diabetes mellitus in several relatives. Examination reveals no significant abnormalities. Dipstick urinalysis is normal. Which of the following is the most appropriate next step in diagnosis?

(A) Blood test for C-peptide
(B) Fasting blood glucose level
(C) Glucose tolerance test
(D) Glycosylated hemoglobin
(E) Urine glucose levels

A

Respuesta: B

The correct answer is B. This patient most likely has type 2 diabetes mellitus. This form represents 90 to 95% of all cases of diabetes mellitus in the U.S. The most common presentation of type 2 is that of an individual found to have hyperglycemia on routine laboratory investigations. Otherwise, patients present most frequently with symptoms due to diabetic complications, such as recurrent vulvovaginitis/balanitis, blurred vision, impotence, and peripheral neuropathy. Type 2 diabetes patients are often obese and older than 30 years. In this example, ophthalmoscopy reveals some of the findings associated with the nonproliferative stage of diabetic retinopathy, which accounts for decreased vision. Fasting blood glucose level is the recommended first-line test to screen for diabetes.

Blood test for C-peptide (choice A) documents function of pancreatic beta cells but is not useful in the diagnosis of diabetes. It may be useful in the diagnosis of insulinoma or factitious insulin injection.

Glucose tolerance test (choice C) is no longer used for screening or diagnostic purposes because of its low specificity. It is performed by measuring glucose levels after the patient is given a load of 75 g of glucose following overnight fasting. It is still useful in the diagnosis of pregnancy-related diabetes.

Glycosylated hemoglobin (choice D) reflects the mean blood glucose levels in the preceding 3 months. Normal values are below 7%. This test is not used for screening or diagnosis, but rather to check the adequacy of long-term glycemic control.

Urine glucose levels (choice E) do not detect hyperglycemia if the blood glucose is below 180 mg/dL (the renal threshold for glucose excretion). Furthermore, this test may be affected by many conditions that can cause false negative or false positive results.

Measurement of urinary protein (choice F) to detect microalbuminuria is the standard test for the diagnosis of early diabetic nephropathy, not diabetes.

How well did you know this?
1
Not at all
2
3
4
5
Perfectly
10
Q

A 61-year-old woman presents with complaints of 2 months of low-grade fevers and malaise. She states that she has been having frequent right-sided headaches without any other associated neurologic symptoms. On physical examination, she has a temperature of 37.9 C (100.2 F), and her neurologic examination is unremarkable. Laboratory results reveal a white blood cell count of 11,200/mm3 and a hematocrit of 36%. Serum electrolytes are normal, and her erythrocyte sedimentation rate (ESR) is 86/min. Which of the following is the most appropriate next step in management?

(A) Carotid artery Doppler flow studies
(B) Ergotamine
(C) High-dose IV penicillin
(D) High-dose steroids
(E) Oral nonsteroidal anti-inflammatory drugs (NSAIDs)

A

Respuesta: D

The correct answer is D. This case describes a typical presentation of temporal arteritis, a systemic disease of inflammation of medium and large arteries. The disease affects older patients (> 55 years) and has a female preponderance. Fever, anemia, elevated ESR, and headaches make up the classic complex of symptoms. Although definitive diagnosis is made by biopsy of the temporal artery, the condition is usually suspected on the basis of the clinical presentation. If steroids are not administered immediately (at the first suspicion of the disease), the patient has a high likelihood of developing blindness. The presentation is not suggestive of narrowing of the carotid artery. If atherosclerosis of the carotid artery were present, the patient may have experienced transient ischemic attacks or stroke, or may have been found to have a bruit over the artery, indicating the need for a carotid artery Doppler flow study (choice A).

Ergotamine (choice B) is used to treat migraine headaches. The ESR would be normal, and the patient afebrile. Migraines typically last 4- 72 hours and may have associated neurologic symptoms. In addition, the age at initial presentation is typically younger. The presentation is not suggestive of a bacterial process. The fever is low grade, and the white cell count is within the normal range.

Temporal arteritis would not respond to antibiotics (choice C). NSAIDs (choice E) are for treatment of headache. The presentation of this case suggests a more systemic disease. Temporal arteritis would not respond to NSAIDs.

How well did you know this?
1
Not at all
2
3
4
5
Perfectly
11
Q

A 55-year-old man comes to medical attention because of a long history of recurrent bouts of deep epigastric pain, associated with elevation of amylase and lipase. He also reports bulky, foul-smelling stools for the past 2 months. A plain x-ray film of the abdomen reveals multiple calcifications in the pancreatic region. At this time, serum levels of amylase and lipase are within normal limits. Which of the following is the most common etiologic factor associated with this condition in the U.S.?

(A) Alcoholism
(B) Biliary stone disease
(C) Cystic fibrosis
(D) Familial predisposition
(E) Hyperlipidemia
(F) Hyperparathyroidism
(G) Malnutrition
(H) Pancreatic neoplasms

A

Respuesta: A

The correct answer is A. The condition described is chronic pancreatitis resulting in malabsorption. Steatorrhea (with greasy, bulky, foul-smelling stools) is a common, although relatively late, manifestation. Chronic alcohol abuse is the etiologic factor accompanying most cases (probably up to 60%) of chronic pancreatitis in the U.S. and other industrialized countries. Alcohol is thought to act by different pathogenetic mechanisms that result in direct injury to acinar cells and blockage of the ducts by inspissated secretion. Loss of parenchyma and fibrosis with calcification develop.

Biliary stone disease (choice B) is associated with a minority of cases. Small gallstones impacted in the distal end of the common bile duct may reverse the flow of pancreatic secretion or allow bile to enter the pancreatic duct.

Cystic fibrosis (choice C) is consistently associated with pancreatic damage, resulting in loss of pancreatic parenchyma and fibrosis but usually presents in childhood or early adulthood. Inflammation plays a minor role, but the ultimate result is similar to chronic pancreatitis, i.e., pancreatic insufficiency and malabsorption.

Familial predisposition (choice D) is seen in a very small percentage of cases associated with an autosomal dominant mutation manifesting with chronic pancreatitis in young age. The mutated gene encodes trypsinogen.

Hyperlipidemia (choice E), types I and V, is one of the rare predisposing conditions for both acute and chronic pancreatitis. The pathogenetic mechanism is uncertain. High serum triglycerides may falsely depress amylase and lipase levels, making the diagnosis of acute pancreatitis in such cases more problematic.

Hyperparathyroidism (choice F) may result in pancreatitis. Hypercalcemia probably triggers inappropriate activation of pancreatic enzymes.

Malnutrition (choice G) is the most common cause of acute and chronic pancreatitis in poor countries of southeast Asia and Africa. These cases are often referred to as tropical pancreatitis, but they are in fact simply related to protein-calorie malnutrition.

Pancreatic neoplasms (choice H) are often associated with some degree of chronic pancreatitis, and it is difficult to determine whether both conditions stem from the same underlying pathogenetic factors or one results from the other.

How well did you know this?
1
Not at all
2
3
4
5
Perfectly
12
Q

A 60-year-old woman presents with complaints of chronic fatigue and mild pruritus. She has a history of rheumatoid arthritis and autoimmune thyroiditis. On examination, her liver is modestly enlarged, firm, and nontender; skin xanthomas are noted. Routine serum chemistry studies show:

Sodium 141 mEq/L
Potassium 5.1 mEq/L
Chloride 102 mEq/L
Bicarbonate 25 mEq/L
Albumin 4.1 g/dL
Urea nitrogen 25 mg/dL
Bilirubin, total 1.3 mg/dL
Creatinine 0.8 mg/dL
AST 55 U/L
ALT 48 U/L
Alkaline phosphatase 240 U/L

Follow-up laboratory studies demonstrate a serum gamma-glutamyl transpeptidase level of 150 U/L, and a serum cholesterol of 240 mg/dL. Immunoglobulin studies reveal a marked elevation of serum IgM. Ultrasound demonstrates diffuse enlargement of the liver without marked echogenicity. Endoscopic retrograde cholangiography demonstrates an intact extrahepatic biliary tree accompanied by stricture and loss of ducts within the liver itself. Liver biopsy shows a florid bile duct lesion with patchy inflammation and destruction of septal and interlobular bile ducts. Antibodies directed against which of the following antigens are present in up to 95% of patients with this disease?

(A) Double-stranded DNA
(B) Hepatitis A virus
(C) Hepatitis B core antigen
(D) Hepatitis C virus
(E) Mitochondria

A

Respuesta: E

The correct answer is E. The disease is primary biliary cirrhosis. This autoimmune disease is characterized by progressive destruction of intrahepatic bile ducts. In early stages, intense inflammation of the bile ducts can be seen on biopsy, often accompanied by bile duct proliferation as the liver attempts to compensate. Later stages are characterized by initial portal fibrosis that eventually evolves to frank cirrhosis. Patients tend to be women, aged 35-70 years, who typically present with insidious disease and often have a history of other autoimmune disease. Chronic fatigue and pruritus are common initial complaints. Hepatomegaly, or hepatosplenomegaly, may be present, as may skin xanthomas or hyperpigmentation. In lab studies, elevations of alkaline phosphatase and gamma-glutamyl transpeptidase are usually out of proportion to those of serum bilirubin and aminotransferases. Endoscopic retrograde cholangiography can be helpful in distinguishing the condition from the related primary sclerosing cholangitis, which damages both the extrahepatic and the intrahepatic biliary system. Biopsies early in the disease may demonstrate florid bile duct destruction; later biopsies are more likely to show nonspecific hepatic fibrosis or cirrhosis. A very helpful test is to measure autoantibodies directed against mitochondrial antigens, since this test is positive in up to 95% of patients with primary biliary cirrhosis; a few patients with “autoimmune” chronic active hepatitis may also have these antibodies.

Anti-double-stranded DNA (choice A) is quite specific for systemic lupus erythematosus.

Hepatitis A (choice B) usually produces acute hepatitis with much more marked elevation of transaminases.

Determination of antibodies to hepatitis B core antigen (choice C) and hepatitis C virus (choice D) is usually used for evaluation of chronic viral hepatitis, which microscopically may show portal inflammation and fibrosis but does not show selective damage to bile ducts.

How well did you know this?
1
Not at all
2
3
4
5
Perfectly
13
Q

A 62-year-old man comes to the physician because of episodic chest pain that manifests as a sensation of precordial tightness, occurs after physical exertion, and is relieved promptly by rest. The patient has noted that the amount of activity sufficient to trigger the pain is relatively constant, such as climbing three flights of stairs or walking uphill for a few minutes. An ECG recorded at rest, however, fails to show any abnormalities. Which of the following is the most appropriate next step in diagnosis?

(A) Ambulatory ECG monitoring
(B) Echocardiography
(C) Exercise ECG
(D) Myocardial perfusion scintigraphy
(E) Coronary arteriography

A

Respuesta: C

The correct answer is C. The patient’s symptomatology is consistent with angina pectoris. During ischemic attacks, ECG usually shows a flat or down-sloping ST depression. The resting ECG may be normal in up to 25% of patients with typical angina between the attacks. In patients without ECG abnormalities at rest, exercise ECG is the most useful and cost-effective test to document myocardial ischemia. Ambulatory ECG monitoring (choice A) is mainly used to document clinically silent (i.e., painless) episodes of myocardial ischemia, which may be more frequent than the clinically apparent ones in some patients.

Echocardiography (choice B) may reveal abnormalities in ventricular wall motion, which can be a result of current ischemia or prior myocardial infarction. It allows the study of left ventricular function, which is an important prognostic factor that influences treatment strategies as well.

Myocardial perfusion scintigraphy (choice D) is performed by injection of radiotracers (thallium-201 and technetium-99m are the most frequently used), which are taken up by viable myocardium. Scintigraphic defects indicate areas of ischemia. This is usually done after exercise ECG.

Coronary arteriography (choice E) is the gold standard for the diagnosis of coronary artery disease since it documents site and severity of stenotic lesions. Generally, it is indicated when coronary artery revascularization is being considered; it is not a first diagnostic procedure. It has a mortality of 1/1000.

How well did you know this?
1
Not at all
2
3
4
5
Perfectly
14
Q

A 49-year-old man with acute pancreatitis develops severe shortness of breath 15 minutes after undergoing placement of a catheter in his subclavian vein. His blood pressure is 100/60 mm Hg, pulse is 124/min, and respirations are 50/min. He is cyanotic and in obvious distress. His neck veins are distended, and his trachea deviates to the left. Breath sounds are diminished on the right side of his chest. Which of the following is the most appropriate next step in management?

(A) Chest x-ray
(B) Removal of the catheter
(C) Endotracheal intubation
(D) Needle thoracostomy in the second right intercostal space
(E) Tube thoracostomy in the left fifth intercostal space

A

Respuesta: D

The correct answer is D. A significant risk associated with catheterization of the subclavian veins is a closed traumatic pneumothorax due to puncture of the apex of the lung. Hypotension, tachycardia, tachypnea, and cyanosis all favor this diagnosis. Classic clues in the patient’s presentation are the distended neck veins, diminished breath sounds on the right side of the chest, and tracheal deviation to the opposite side. The most appropriate immediate treatment is needle thoracostomy at the second right intercostal space followed by chest tube insertion at the right fifth intercostal space.

Chest x-ray (choice A) is not necessary in this patient since the clinical examination was sufficient for making the diagnosis. Waiting for chest x-ray results before treating this unstable patient could prove fatal. Note, however, that chest x-rays are routinely performed after catheterizations to rule out subclinical pneumothoraces. On x-ray films, a pneumothorax appears as a region of air without peripheral lung markings limited by a distinct pleural boundary with medial lung markings.

Removal of the catheter (choice B) would not treat the punctured lung. Note that future attempts at central line placement should be attempted on the right side in this patient to avoid the possibility of creating bilateral pneumothoraces.

Endotracheal intubation (choice C) would not relieve the pneumothorax and would not be expected to improve respiratory status until the pneumothorax was successfully treated.

Tube thoracostomy in the left fifth intercostal space (choice E) would be on the wrong side in this patient with a right pneumothorax.

How well did you know this?
1
Not at all
2
3
4
5
Perfectly
15
Q

A 31-year-old Asian American man presents to the clinic for an annual physical. He has a 19-year history of type 1 diabetes, requiring 10 units NPH insulin each morning and 8 units NPH in the evening, with frequent blood glucose checks and regular insulin dosing throughout the day. He does not keep a log of his blood glucose values. A urine dipstick test shows 2+ albumin. His hemoglobin A1c (HbA1c) is 7.9%. Which of the following is the most appropriate next step in management to prevent morbidity?

(A) Add a standing regular insulin dose at lunchtime
(B) Begin ACE inhibitor therapy
(C) Discuss options for using an insulin pump
(D) Increase his morning NPH insulin dose
(E) Send a 24-hour urine collection specimen for total protein

A

Respuesta: B

The correct answer is B. The concepts underlying this question are those of diabetes mellitus and the prevention of its complications. Many clinical trials have shown the beneficial effects of ACE inhibitors on preventing nephropathy and slowing the progression of established nephropathy in diabetics. This patient has microalbuminurial as shown by his urine dipstick, suggesting developing renal disease. It is the standard of care that all diabetics be given an ACE inhibitor if they are able to tolerate its blood pressure effects.

Adding a standing regular insulin dose at lunchtime (choice A) or increasing his morning NPH insulin dose (choice D) may be appropriate, although we do not know the details of his daily glucoses since he does not keep a glucose log. His non-optimal HbA1c clearly indicates that his blood glucose control needs to be improved, but at this time we do not yet know the best way to accomplish this goal. The standard of care is for a goal HbA1c less than 7%.

Discussing options for using an insulin pump (choice C), similar to choices A and D, may be an option, depending on the details of this patients daily blood glucoses and his ability to comply with a stricter regimen.

Sending a 24-hour urine collection specimen for total protein (choice E) is not appropriate at this stage given the likelihood that this patient is developing diabetic nephropathy. Quantifying urine protein will not change this patient’s management, namely, the addition of an ACE inhibitor.

How well did you know this?
1
Not at all
2
3
4
5
Perfectly
16
Q

A 55-year-old patient presents with chronic cough. In addition to the cough, the patient has gained weight recently with development of a “buffalo hump” and Cushingoid features. A chest x-ray film demonstrates a mass involving the central area of the chest. Bronchoscopy is performed, and it proves possible to biopsy the tumor during the procedure. Which of the following is the most likely diagnosis?

(A) Adenocarcinoma
(B) Bronchioloalveolar carcinoma
(C) Large cell carcinoma
(D) Small cell carcinoma
(E) Squamous cell carcinoma

A

Respuesta: D

The correct answer is D. The patient has bronchogenic lung cancer, which has produced Cushing syndrome as a paraneoplastic syndrome related to secretion of substances similar to ACTH. Small cell carcinoma of the lung is particularly notorious as a secretor of bioactive substances, including ADH, ACTH, parathormone, prostaglandins, calcitonin, gonadotropins, and serotonin. Small cell carcinoma of the lung has a strong association with smoking.

Adenocarcinoma of the lung (choice A), when used in questions, usually refers to adenocarcinoma without further differentiating features. This form of cancer can be seen in bronchi, as a coin lesion in the lung periphery, or involving scarred areas of lungs.

Remember bronchioloalveolar carcinoma (choice B) as the type that is not associated with smoking.

Large cell carcinoma (choice C) is an aggressive, undifferentiated form of lung cancer.

Associate squamous cell carcinoma of the lung (choice E) with the specific paraneoplastic syndrome of hypercalcemia.

How well did you know this?
1
Not at all
2
3
4
5
Perfectly
17
Q

A 61-year-old man is hospitalized after receiving an implantable cardiac defibrillator (ICD). The patient has a long history of coronary disease and sustained an anterior wall myocardial infarction 3 years ago. Two weeks ago, he had an episode of pulseless ventricular tachycardia and was successfully resuscitated. This episode led to the ICD placement. In addition to the ICD, the cardiologist also plans to initiate antiarrhythmic therapy with amiodarone. Which of the following is the most important side effect of this therapy?

(A) Hypotension
(B) Pulmonary fibrosis
(C) Prolongation of the QT interval
(D) Recurrent ventricular arrhythmia
(E) Skin discoloration

A

Respuesta: B

The correct answer is B. Amiodarone is a class III antiarrhythmic agent with many electrophysiologic, as well as a number of potential, side effects. The most feared side effect, causing the greatest amount of long-term morbidity and mortality, is pulmonary fibrosis, occurring in up to 17% of patients in some series (average 10%). The incidence is clearly related to the total daily dosing, which is taken into account when the drug is prescribed in the U.S.

Hypotension (choice A) is common with the IV formulation of this drug but, once past the loading dose, is usually not an issue.

Prolongation of the QT interval (choice C) is very common with amiodarone therapy and could lead to a polymorphic ventricular arrhythmia termed “torsades de pointes.” However, the incidence of torsades among patients on amiodarone with prolonged QTs is very low, less than 1%.

Recurrent ventricular arrhythmia (choice D) is a problem for any patient who has had a previous arrhythmia. In theory, amiodarone at therapeutic levels will decrease, but not eliminate, the possibility of recurrence.

Skin discoloration (choice E) is a common occurrence with this medication but, despite its cosmetic appearance, is not a significant issue for the patient.

How well did you know this?
1
Not at all
2
3
4
5
Perfectly
18
Q

A 32-year-old woman is referred to a neurologist for evaluation of unsteady gait and numbness in the right foot. Examination reveals weakness of the right lower extremity with muscle spasticity and decreased vibratory sensation. MRI studies show cerebral and spinal cord changes suspicious for demyelinating lesions. A lumbar puncture is performed for examination of CSF. Which of the following CSF findings would be most consistent with a diagnosis of demyelinating disorder?

(A) Elevated protein with marked lymphocytosis
(B) Elevated protein with normal cell count
(C) Marked neutrophilic leukocytosis with reduced glucose
(D) Mildly increased protein with oligoclonal IgG bands
(E) Normal protein with mild lymphocytosis

A

Respuesta: D

The correct answer is D. Clinical symptomatology and MRI findings constitute the mainstay for the diagnosis of multiple sclerosis, but CSF examination may add confirmatory evidence. An abnormality frequently found in multiple sclerosis is the presence of oligoclonal bands of IgG detected by CSF electrophoresis. This appears to result from activation of lymphocytic subsets directed against specific white matter antigens, such as myelin basic protein. CSF cell count may be normal or slightly elevated.

Elevated protein with marked lymphocytosis (choice A) is usually associated with viral or mycobacterial meningitis/meningoencephalitis. CSF glucose is normal in viral infections but is reduced in mycobacterial infections.

Elevated protein with a normal cell count (choice B) is characteristic (but not pathognomonic) of Guillain-Barré syndrome.

Marked neutrophilic leukocytosis with reduced glucose (choice C) is highly characteristic of bacterial meningitis. Elevated CSF protein would also be present.

Normal protein with mild lymphocytosis (choice E) is nonspecific and may develop as a reaction (neighborhood reaction) to intracranial processes, such as abscess, mastoiditis, and tumor.

How well did you know this?
1
Not at all
2
3
4
5
Perfectly
19
Q

A 70-year-old man is brought to the emergency department by his family because of the rapid onset of rightsided weakness and confusion. On arrival, the patient is drowsy, but examination confirms a right hemiparesis associated with left deviation of the eyes. The patient is admitted with a preliminary diagnosis of cerebral infarction. A CT scan of the head performed 12 hours following symptom onset reveals changes consistent with ischemic necrosis in the territory of the middle cerebral artery. The patient’s neurologic status deteriorates rapidly on the third hospital day, and he lapses into coma. He dies the next day. An autopsy confirms a large infarct in the territory of the middle cerebral artery, associated with massive swelling of the left hemisphere, transtentorial herniation, and pontine Duret hemorrhages. Which of the following treatments might have prevented such an outcome?

(A) Anticoagulants
(B) Barbiturates
(C) Calcium channel blockers
(D) Corticosteroids
(E) NMDA receptor antagonists

A

Respuesta: D

The correct answer is D. The edema (swelling) that develops around a cerebral infarct becomes particularly pronounced 48-72 hours following ischemic necrosis and may be of massive proportions if infarction is extensive. This leads to increased intracranial pressure, often resulting in cerebral herniations. Duret hemorrhages along the midline of the pons are secondary to transtentorial (uncal) herniation. Corticosteroids, such as prednisone (up to 100 mg/day) and dexamethasone (16 mg/day), are used to reduce cerebral edema following brain infarction and prevent herniation syndromes.

Anticoagulants (choice A) are not used in the acute management of stroke unless there is a proven source of thromboemboli, such as cardiac disease (e.g., atrial fibrillation and mitral valve prolapse). Heparin is the drug of choice. Anticoagulation can be started only if intracerebral hemorrhage has been ruled out by CT. Several compounds that may minimize death of hypoxic neurons in the “penumbra” of the infarcted region have been used or are under investigation.

These compounds include barbiturates (choice B), which diminish neuronal metabolism and energy requirements; calcium channel blockers (choice C), such as nimodipine, which have been shown to reduce neurologic deficits from stroke; and drugs that block receptors for glutamate, such as NMDA receptor antagonists (choice E), which seem to reduce infarct extent in experimental models.

How well did you know this?
1
Not at all
2
3
4
5
Perfectly
20
Q

A 60-year-old woman is hospitalized with a severe case of pneumonia that is treated with cephalosporins. Her condition improves over the course of therapy, and after 6 days, she develops loose stools that progresses to frequent bouts of foul-smelling, watery diarrhea with small amounts of blood over the course of 10 hours. There is associated abdominal pain, nausea and vomiting, fever, leukocytosis, and hypotension. Physical examination reveals diffuse pain on palpation, and an abdominal x-ray shows marked dilatation of the transverse colon and mucosal edema. Sigmoidoscopy is performed and reveals diffuse pseudomembranes over the colon that reveal an erythematous, inflamed muscosa when removed. Which of the following is this patient at greatest risk for?

(A) Colon cancer
(B) Colonic perforation
(C) Fistula formation
(D) Gangrenous necrosis
(E) Hemorrhage
(F) Malabsorption
(G) Vitamin deficiency

A

Respuesta: B

The correct answer is B. This woman has antibioticassociated pseudomembranous colitis, which has resulted in toxic megacolon, a known complication of severe pseudomembranous colitis. The causative organism of this colitis is Clostridium difficile, a gram- positive, spore-forming, anaerobic bacillus that overgrows in the setting of broad-spectrum antibiotic use, especially clindamycin, amoxicillin, ampicillin, or cephalosporins. Symptoms range from loose stools in the mildest cases to toxic megacolon (fever, nausea, vomiting, ileus) and colonic perforation (rigid abdomen, rebound tenderness) in the most severe cases. In most cases, the diagnosis can be made on clinical grounds without the need for colonoscopy. If colonoscopy is performed, it may show either a nonspecific colitis or, in severe cases, pathognomonic pseudomembranes. Many cases occur after antibiotic use (of up to 6 weeks). Mild cases resolve with the discontinuation of antibiotics, but severe cases require treatment with oral metronidazole (first line of therapy) or oral vancomycin (in resistant cases) to prevent toxic megacolon and resultant colonic perforation. Two thirds of patients with toxic megacolon require surgical intervention.

Colon cancer (choice A) is not associated with pseudomembranous colitis. There is a significantly higher risk in patients who have had ulcerative colitis for more than 10 years, a family history of colon cancer, or a history of multiple colonic polyps.

The formation of fistulas (choice C) and colonic hemorrhages (choice E) are complications of diverticular disease. This patient’s condition was not diverticulitis, and because this patient did not show evidence of diverticular disease on colonoscopy, it is unlikely that either of these complications will occur.

Gangrenous necrosis (choice D) is not a complication of pseudomembranous colitis. It may occur as a complication of mesenteric vessel occlusion. Typically, gangrenous necrosis is found in a patient with an underlying cause of embolism or atherosclerosis, and there is often a history of abdominal pain after eating (mesenteric angina). However, there is no association with antibiotic use, and the presence of pseudomembranes on endoscopy confirms a different diagnosis.

Malabsorption (choice F) and vitamin deficiency (choice G) are not complications of pseudomembranous colitis. Furthermore, they would unlikely be complications in conditions affecting the colon because vitamin and nutrient absorption is a function of the small, not large, intestine.

How well did you know this?
1
Not at all
2
3
4
5
Perfectly
21
Q

A 35-year-old man with a history of chronic hepatitis B presents to the emergency department with severe abdominal pain. The patient states that he has had intermittent fevers and a 15-lb weight loss over the past 6 months. His temperature is 38 C (100.4 F), blood pressure is 160/120 mm Hg, pulse is 60/min, and respirations are 15/min. Abdominal examination demonstrates guarding and diffuse pain. Examination of his legs is remarkable for purpura and several distinctive pea-sized nodules along the course of superficial arteries. The erythrocyte sedimentation rate is 30 mm/hr. Urinalysis reveals proteinuria, hematuria, and proteinaceous casts. Biopsy of one of the pea-shaped lesions of the leg would likely show which of the following in vessels walls?

(A) Atherosclerotic plaques
(B) Fungal hyphae
(C) Giant cells
(D) Granulomas
(E) Neutrophils

A

Respuesta: E

The correct answer is E. Polyarteritis nodosa (PAN) is a vasculitis of medium-sized arteries that can occur at any age and has a 2:1 male predominance. An important feature of the vasculitis is its spotty distribution, with individual lesions involving typically only 1 cm or shorter lengths of blood vessel. Some patients have hepatitis B antigenemia, and it is suspected that circulating immune complexes related to the chronic hepatitis may trigger the vasculitis. Diagnosis can be difficult because a variety of organ systems may be affected. Most patients have nonspecific constitutional findings, including hypertension, weight loss, fever, and elevated erythrocyte sedimentation rate (a marker for chronic disease with an immune component). Many patients seek medical attention because of an acute abdomen or gastrointestinal bleeding. Other presentations involving other systems, such as myocardial infarction, progressive renal failure, or infarction of individual abdominal organs, can occur. Urinalysis may be helpful in that the findings may suggest the possibility of multisystem disease. Particularly helpful, if present, are distinctive pea-shaped nodules along the superficial arteries of the legs. The nodules may be accompanied by vasculitic purpura, urticaria, or even subcutaneous hemorrhage with gangrene. Microscopically, the acute lesions of PAN show fibrinoid necrosis and neutrophil infiltration into the vessel wall. With healing, the infiltrating cell population shifts to macrophages and plasma cells. In the healed lesion, residual damage in the form of vessel wall fibrosis and fragmentation of the internal elastic membrane can be seen.

Atherosclerotic plaques (choice A) severe enough to cause the patient’s symptoms would be unusual in a 35-year-old man. Atherosclerotic plaques most often involve the aorta and the more proximal regions of its branches.

Associate fungal hyphae (choice B) with Aspergillus, or less frequently with Candida, both of which can destroy blood vessels, most often in patients with underlying immunosuppression.

Associate giant cells (choice C) with giant cell (temporal) arteritis, which most commonly involves the arteries of the head, notably the temporal and ophthalmic arteries.

Associate granulomas (choice D) of blood vessels with giant cell arteritis, Churg-Strauss syndrome, and Wegener granulomatosis.

22
Q

A 22-year-old woman presents with a 1-week history of an itchy rash, which manifested a few weeks after coming back from a trip with her friends. One of the friends has developed a similar rash. The patient reports that the itching keeps her awake at night. Physical examination reveals a linear papulovesicular eruption along the waistline, axillary folds, and finger webs. Linear burrows are evident on close inspection. Which of the following is the most appropriate next step in diagnosis?

(A) Microscopic examination of skin scrapings obtained after placing oil on lesions
(B) Microscopic examination of skin scrapings treated with potassium hydroxide
(C) Measurement of serum IgE levels
(D) Allergen challenge tests
(E) Skin biopsy

A

Respuesta: A

The correct answer is A. The symptomatology and findings on physical examination are strongly suggestive of scabies, due to infestation by Sarcoptes scabiei. This mite is acquired by direct contact with an infected person and penetrates into the skin, producing burrows that are visible on close (or hand lens) examination. The areas of the body most frequently affected include the axillary and genital regions, waistline, and finger webs. Pruritus tends to be particularly intense at night. The most effective way to confirm a diagnosis of scabies is to place a drop of mineral oil on a suspected area (one of the burrow holes), then unroof the burrow hole with a curette ring and obtain scrapings that are examined under a microscope.

Microscopic examination of skin scrapings treated with potassium hydroxide (choice B) is the method of choice to demonstrate fungal organisms in skin lesions. To perform this test, skin scrapings are obtained by using a ring curette or a #15 blade. The scrapings are placed on a glass slide and treated with a drop of 10 to 20% KOH. After a few minutes, a cover slip can be applied on the glass slide, and the preparation examined under the microscope or a magnifying lens. Hyphae and spores can be identified. KOH treatment is necessary to dissolve the keratin.

Measurement of serum IgE levels (choice C) and allergen challenge tests (choice D) are appropriate investigations when an allergic disorder is suspected. Clinical history and physical examination clearly indicate that this is not the case.

Skin biopsy (choice E) is an invaluable investigative tool that should be resorted to when the clinical approach (i.e., history and objective findings) is insufficient to reach a diagnosis. Scabies can be easily diagnosed without resorting to biopsy.

23
Q

A 35-year-old man being treated with phenytoin for epilepsy comes to the physician for a routine check-up examination. He has been seizure-free for the past year. Physical examination reveals pallor of skin and mucosae, slightly jaundiced discoloration of sclerae, and a red and shiny tongue. He denies paresthesias, and sensation is normal on neurologic examination. Significant results of blood and serum studies include:

Hematocrit 28%
Hemoglobin 8.5 g/dL
Mean corpuscular volume 130 fL
Reticulocytes 0.2%
Platelets 140,000/mm3
Leukocytes 4800/mm3
Bilirubin Total 2.0 mg/dL
Direct Bilirrubin 0.3 mg/dL
Lactate dehydrogenase 600 U/L

A peripheral blood smear reveals macrocytes, ovalocytes, and hypersegmented neutrophils. Which of the following is the most likely cause of this patient’s anemia?

(A) Autoimmune hemolysis
(B) Bone marrow aplasia
(C) Folate deficiency
(D) Iron deficiency
(E) Vitamin B12 deficiency

A

Respuesta: C

The correct answer is C. This patient’s anemia is clearly of the macrocytic/megaloblastic type, as evidenced by strikingly high mean corpuscular volume (MCV). Hypersegmented neutrophils also represent a characteristic accompanying feature. Glossitis (with red and shiny tongue) is another typical manifestation. The key to the correct answer is the history of treatment with phenytoin. This drug acts as an antimetabolite of folic acid and may induce folic acid- deficiency anemia even in the presence of adequate dietary intake. Besides drug-induced cases, the most frequent forms of folic acid deficiency result from inadequate diet with insufficient amounts of fresh fruits and vegetables. Alcoholics and anorexic patients are at increased risk.

Autoimmune hemolysis (choice A) is due to reduced survival of red blood cells resulting from antibodies to erythrocyte antigens. In this condition, reticulocytosis is prominent, since the bone marrow maintains the ability to compensate for the red blood cell loss. Unconjugated hyperbilirubinemia is a feature present in all hemolytic anemias and, to some extent, in megaloblastic anemia. However, the striking increase in MCV is characteristic of megaloblastic anemia, whereas hemolytic anemias are associated with normal MCV.

Bone marrow aplasia (choice B) will lead to severe anemia (aplastic anemia) accompanied by neutropenia and thrombocytopenia. The disorder results from injury to the marrow precursors of myeloid, erythroid, and megakaryocytic lines. It may be idiopathic or associated with immunologic conditions, radiation, toxins, and drugs (among the latter, phenytoin itself).

Anemia due to bone marrow depression is normocytic and normochromic. Iron deficiency (choice D) results in microcytic anemia, with MCV values often less than 80 fL. The most common cause of iron- deficiency anemia in the U.S. is chronic blood loss secondary to an underlying disorder, e.g., menstrual loss in women and gastrointestinal malignancies in both men and women.

Vitamin B12 deficiency (choice E) causes macrocytic anemia identical to that secondary to folic acid deficiency. However, vitamin B12 deficiency also results in neurologic deficits, such as paresthesias, ataxia, and decreased vibration and position sense. The most common cause of vitamin B12 deficiency in the U.S. is pernicious anemia due to autoimmune-related atrophic gastritis.

24
Q

A 35-year-old man comes to the emergency department because he is experiencing palpitations and is afraid he is having a heart attack. On questioning, the patient has had a number of these episodes in the past few months, although the present one is the worst. His temperature is 37 C (98.6 F), blood pressure is 170/135 mm Hg, pulse is 140/min, and respirations are 17/min. Physical examination is remarkable only for diaphoresis. Within the hour, before definitive therapy is begun, the patient’s blood pressure drops spontaneously to 140/85 mm Hg, and his symptoms improve. Measurement of which of the following in a 24-hour urine collection is most likely to confirm the probable diagnosis?

(A) 17-Hydroxycorticosteroids
(B) Porphobilinogen
(C) Pregnanediol
(D) Tetrahydrocortisol
(E) Vanillylmandelic acid

A

Respuesta: E

The correct answer is E. Pheochromocytomas are rare neoplasms of the adrenal medulla (or of extra-adrenal sites) that produce epinephrine and/or norepinephrine, thereby causing episodic or continuous hypertension. Since the epinephrine and norepinephrine secretion tends to be episodic, plasma levels of these hormones may be normal. Therefore, it has become customary to look for metabolites of epinephrine and norepinephrine in 24-hour urine collections, thereby averaging out the periodicity. The specific compounds analyzed are the metanephrines, vanillylmandelic acid (VMA), and homovanillic acid (HVA).

17-Hydroxycorticosteroids (choice A) include cortisol, cortisone, and 11-deoxycortisol. These steroid hormones are unrelated to the amino-acid-derived structure of epinephrine and norepinephrine.

Porphobilinogen (choice B) is a porphyrin precursor that is elevated in some of the porphyrias.

Pregnanediol (choice C) is a metabolite of the steroid hormone progesterone.

Tetrahydrocortisol (choice D) is a derivative of the steroid cortisol that is found in urine.

25
A 55-year-old white man presents to the emergency department with severe abdominal pain that has been radiating through to his back for the past 2 days. The man appears acutely ill and is sitting in a markedly bentover position, holding his arms over his abdomen. The patient states that this posture makes him feel slightly better, and comments that sudden movements and deep breathing make the pain worse. He has also been throwing up, even when there is nothing in his stomach. He describes the pain as “terrible” and begs for narcotics. On questioning, he admits to heavy drinking for the past 5 or 10 years. He denies use of street drugs. His temperature is 38.1 C (100.6 F), blood pressure is 85/60 mm Hg when lying down and 60/45 mm Hg when sitting, pulse is 120/min and regular, and respirations are 22/min and shallow. His lungs are clear to auscultation. Pressure on the upper portion of the abdomen intensifies the pain. Examination of the extremities reveals multiple bruises but no needle marks. The liver edge can be felt and has a nodular character. A complete blood count shows an erythrocyte count of 3.5 million/mm3 , white blood cell count of 18,000/mm3 with predominately neutrophils and increased band forms, and a platelet count of 200,000/mm3 . Which of the following is the most appropriate next step in diagnosis? (A) Blood urea nitrogen (B) Chest x-ray (C) Serum amylase level (D) Serum transaminase levels (E) Esophagogastroduodenoscopy
Respuesta: C The **correct answer is C**. This case is a fairly classic presentation of acute pancreatitis. These patients often have a history of either alcohol abuse or gallstone disease. Alcohol intake greater the 100 g/day favors precipitation of protein of the pancreatic enzymes within the ducts, which eventually (over a period of years) cause enough blockage to trigger pancreatitis. The pain described is typical of acute pancreatitis, but some patients with more chronic disease may have surprisingly little pain. Very early in the disease, the patient’s temperature may be subnormal, but fever usually develops within a few hours of onset. An elevated white count is usually present. The most helpful laboratory tests are serum amylase and serum lipase, elevation of which are considered strongly supportive of a diagnosis of pancreatitis. Blood urea nitrogen **(choice A)** is a marker for renal dysfunction. A chest x-ray film **(choice B)** might show pneumonia but would probably not be helpful in this patient with clear lungs. Serum transaminase levels **(choice D)** may very well be elevated in this alcoholic patient, but his most significant acute problem is his pancreatitis, not his probably damaged liver. Esophagogastroduodenoscopy **(choice E)** might show peptic ulcer disease, but severe pain would usually be seen only with perforation, and the patient would probably be vomiting blood in that setting.
26
A 40-year-old woman presents to the physician because of increased nervousness for the past 3 months. She reports insomnia, frequent palpitations without an identifiable cause, and weakness. She has no significant past medical history and takes no medications. Vital signs are remarkable for a blood pressure of 150/60 mm Hg and a pulse of 135/min. She appears anxious, and despite being in the middle of winter, is dressed in a t-shirt and shorts. Physical examination reveals proptosis and eyelid retraction, moist skin, mild hand tremor, and a palpable diffuse goiter. Which of the following is the most likely diagnosis? (A) Euthyroid sick syndrome (B) Follicular carcinoma of the thyroid (C) Graves disease (D) Hashimoto thyroiditis (E) Subacute thyroiditis
Respuesta: C The **correct answer is C**. This patient is exhibiting the classic presentation of hyperthyroidism. Other signs and symptoms that may be seen include atrial fibrillation, nervousness, increased appetite, weight loss, frequent bowel movements, diplopia, conjunctival injection, and pretibial myxedema. The most common cause of hyperthyroidism is Graves disease (diffuse toxic goiter); in fact, infiltrative ophthalmopathy (as well as pretibial myxedema) is a specific autoimmune manifestation of Graves disease. The disease has an autoimmune basis, with antibodies directed against the TSH receptors, causing continuous thyroid gland stimulation. Euthyroid sick syndrome **(choice A)** causes asymptomatic thyroid hormone alterations, usually in patients with serious systemic disease. Follicular carcinoma of the thyroid **(choice B)** usually does not cause hyperthyroidism. Hashimoto thyroiditis **(choice D)** can cause transient hyperthyroidism early in its course, but this is uncommon compared with Graves disease. Subacute thyroiditis **(choice E)** usually causes striking thyroid gland tenderness and may cause transient hyperthyroidism.
27
A malnourished and dehydrated 54-year-old man with a history of alcoholism is brought to the emergency department. His temperature is 36.9 C (98.4 F), blood pressure is 105/65 mm Hg, pulse is 98/min, and respirations are 14/min. Neurologic assessment reveals decreased reflexes and sensation in the legs, and an ataxic gait. On admission, laboratory tests show: Albumin 2.9 g/dL Sodium 105 mEq/L Potassium 3.9 mEq/L Chloride 101 mEq/L Plasma osmolality 256 mOsm/kg Na fractional excretion < 0.5% He is treated with parenteral administration of 100 mg of thiamin and IV infusion of isotonic saline and glucose. Twelve hours after treatment is started, the patient becomes quadriplegic and mute, although he appears to be able to communicate by slow eye blinking. A blood sample at this time shows the following values: Sodium 126 mEq/L Potassium 4.0 mEq/L Chloride 105 mEq/L Despite adequate therapeutic interventions, the patient dies on the third hospital day. Which of the following postmortem findings will most likely be obtained at autopsy? (A) Alzheimer type 2 glia, consistent with hepatic encephalopathy (B) Central pontine myelinolysis (C) Cerebellar vermis atrophy (D) Subacute combined degeneration of the spinal cord (E) Wernicke encephalopathy
Respuesta: B The **correct answer is B**. The clinical picture is consistent with a “locked-in” state that can be caused by central pontine myelinolysis. This condition consists of acute demyelination of the basilar pons, resulting in interruption of the corticobulbar (excluding the fibers to cranial nerve nuclei I-IV) and corticospinal tracts with preservation of sensory input. The patient can close his eyes by inhibition of the levator palpebrae muscle, innervated by the oculomotor nerve. One of the most common causes of this disorder is excessively rapid correction of hyponatremia. To prevent this complication, it is recommended that the increase in serum sodium level should not exceed 1 mEq/L/hr or 25 mEq/L within the first day of therapy. Alzheimer type 2 glia **(choice A)** refers to modified astrocytes that develop in patients with chronic liver failure. Hepatic encephalopathy is certainly a consideration in an alcoholic patient, but it would manifest with mental status changes and characteristic flapping tremor (asterixis) without paralysis. Cerebellar vermis atrophy **(choice C)** may be due to chronic alcoholism and is responsible for the ataxic gait frequently seen in chronic alcoholics. Subacute combined degeneration of the spinal cord **(choice D)** results in ataxia, numbness, and spastic paresis of the lower extremities. Wernicke encephalopathy **(choice E)** manifests with nystagmus, ophthalmoparesis, and confusion. Both disorders are related to thiamin deficiency.
28
A 64-year-old man is brought to the emergency department because of sudden onset of tearing chest pain that seems to originate in the anterior chest and radiate to the back in the interscapular region. A few weeks ago, he had a dental abscess, which resolved with extraction followed by a full course of ampicillin. On examination, he is oriented to person, place, and time; however, he appears in acute distress, is diaphoretic, and is breathing with difficulty. His temperature is 37.1 C (98.8 F), blood pressure is 174/68 mm Hg, pulse is 206/min, and respirations are 25/min. There is no jugular vein distention or hepatomegaly. Auscultation reveals a diastolic murmur along the left sternal border. The lungs are clear to auscultation. The ECG shows no signs of myocardial ischemia. A chest x-ray film demonstrates widening of the mediastinum. Which of the following is the most likely diagnosis? (A) Aortic dissection (B) Acute mediastinitis (C) Acute pericarditis (D) Cardiac tamponade (E) Myocardial infarction
Respuesta: A The **correct answer is A**. A tearing, excruciating chest pain that radiates to the back should always generate the clinical suspicion of aortic dissection involving the aortic arch. The patient presents with extreme signs of distress. Aortic insufficiency, with its associated diastolic murmur and widened pulse pressure, frequently develops. A discrepancy in blood pressure or pulse between the right and left arms is an additional supporting sign. Mediastinal widening is often seen on chest x-ray, but the diagnosis should be confirmed by CT or MRI scans. Hypertension is the most common predisposing factor, but Marfan syndrome is a classic condition associated with aortic dissection. Acute mediastinitis **(choice B)** is a rare infectious complication due to extension of suppurative processes from adjacent cervical organs (e.g., peritonsillitis, thyroiditis) or perforation of the esophagus or trachea. The patient has chest pain but lacks fever and other systemic signs of infections. The history of a recent dental abscess should not deceive you. Acute pericarditis **(choice C)** produces chest pain, which is gradual in onset and usually accompanied by a friction rub. If pericardial effusion is particularly abundant, cardiac tamponade **(choice D)** may ensue. The latter will result in acute signs and symptoms of cardiac failure, necessitating emergency pericardiocentesis to relieve the pressure on the heart. Myocardial infarction **(choice E)** is probably the most important differential diagnosis to consider in case of aortic dissection, but the absence of ECG changes suggesting myocardial ischemia argues against it in this patient.
29
A 25-year-old man is admitted to a burn unit after an automobile accident in which he lost consciousness and then was burned over 65% of his body. Two days after his accident, the nurse notices that small amounts of blood are returned through his nasogastric tube when she checks it prior to administering fluids. Endoscopy shows multiple, tiny to small (2-20 mm in diameter) ulcers in the corpus of the stomach. Which of the following is the most likely diagnosis? (A) Acute erosive gastritis (B) Chronic erosive gastritis (C) Gastric atrophy (D) Nonerosive gastritis (E) Superficial gastritis
Respuesta: A The **correct answer is A**. This is acute erosive gastritis, or “stress” gastritis, which is important to diagnose early because it can cause fatal gastrointestinal bleeding. Patients who develop this condition are usually already severely ill and very vulnerable to the problems of acute hemorrhage. Risk factors include severe burns, CNS trauma, sepsis, shock, and other organ system failure (respiratory, liver, renal, or multi-organ). Less severely ill patients may develop acute erosive gastritis as a complication of drug therapy (particularly, NSAIDs) or alcohol use. The pathogenesis of acute erosive gastritis in severely ill patients is thought to involve decreased mucosal defense mechanisms and localized mucosal ischemia, possibly compounded by acid hypersecretion (particularly in burns, CNS trauma cases, and sepsis), which promotes mucosal damage. Because of the potential seriousness of the condition, most intensive care units use measures such as early enteral feeding, IV H2 blockers, or antacids to prevent it. Chronic erosive gastritis **(choice B)** can be seen with drugs (aspirin and NSAIDs), Crohn disease, and viral infections. It is characterized by punctate lesions on the ridges of thickened rugal folds by endoscopy. Gastric atrophy **(choice C)** is a nonerosive lesion of the stomach with mucosal atrophy that can complicate long-standing gastritis of various etiologies. It can also occur in connection with autoantibodies to parietal cells, producing pernicious anemia. Nonerosive gastritis **(choice D)** is related to Helicobacter pylori infection characterized by inflammation through the entire mucosa. Superficial gastritis **(choice E)** is a nonerosive, and often asymptomatic, mild form of gastritis related usually to H. pylori infection.
30
A 56-year-old woman with a long history of untreated hypertension is brought to the emergency department because of severe headache and confusion. The patient is oriented to person, but not to time or place. Her blood pressure is 230/140 mm Hg, pulse is 86/min, and respirations are 18/min. Funduscopic examination reveals optic disk edema, and a dipstick test shows protein in the urine. Which of the following is the most appropriate pharmacotherapy? (A) Clonidine (B) Enalaprilat (C) Esmolol (D) Furosemide (E) Hydralazine (F) Nifedipine (G) Nitroglycerin (H) Sodium nitroprusside
Respuesta: H The **correct answer is H**. The patient is exhibiting the clinical picture of hypertensive emergency, in which striking elevation of blood pressure (diastolic values often >130 mm Hg) is associated with signs and symptoms of both cerebral damage (headache, confusion, and optic disk edema) and renal damage (proteinuria). Aggressive management is required to avoid serious complications or death and is aimed at gradually lowering blood pressure within 1 hour. IV infusion of sodium nitroprusside is the treatment of choice, but its administration should be carefully titrated to obtain the desired effect without excessively rapid reduction in blood pressure. Clonidine **(choice A)** and nifedipine **(choice F)** are oral antihypertensive agents that should be reserved for less severe cases of hypertension (so-called hypertensive urgencies), in which signs of cerebral or renal damage are not detectable. Enalaprilat **(choice B)** is the active form of enalapril. Its antihypertensive action is delayed; thus, it should be used only in conjunction with other, faster-acting agents. Esmolol **(choice C)** is a beta blocker drug that has been useful in treating hypertensive emergencies in the presence of myocardial ischemia. However, it should be combined with some other antihypertensive agent. Furosemide **(choice D)**, as well as other IV loop diuretics, may be useful in the presence of signs of cardiac failure or fluid retention, but their action is slow to manifest. Hydralazine **(choice E)** is mainly used in hypertensive crises affecting children and pregnant women. It may induce dangerous reflex tachycardia. Nitroglycerin **(choice G)**, administered by IV infusion, is less effective than nitroprusside but is useful in patients with manifestations of myocardial ischemia.
31
A 29-year-old man consults a physician because he has developed chronic weakness and fatigue. He also feels dizzy when he first stands up in the morning. On further questioning, he reports cold intolerance and frequent urination. Physical examination is notable for facial puffiness; coarse, sparse, hair; and a hoarse voice. Laboratory studies show: Sodium 125 mEq/L Potassium 5.5 mEq/L Bicarbonate 25 mEq/L Urea nitrogen (BUN) 22 mg/dL Serum glucose 380 mg/dL Thyroid-stimulating hormone (TSH) 9.5 IU/mL Which of the following is the most likely diagnosis? (A) Multiple endocrine neoplasia (MEN), type I (B) MEN, type IIA (C) MEN, type IIB (D) Polyglandular deficiency syndrome, type I (E) Polyglandular deficiency syndrome, type II
The correct answer is E. The polyglandular deficiency syndromes are autoimmune disorders that cause subnormal functioning of several endocrine glands concurrently. This patient has the type II variant, which has peak incidence at age 30 and always involves the adrenal cortex. Thyroid and pancreatic islet involvement, producing type 1 diabetes mellitus, are also common. In this patient, adrenocortical insufficiency is suggested by the cluster of serum sodium < 130 mEq/L, serum potassium > 5 mEq/L, plasma bicarbonate < 28 mEq/L, and BUN >20 mg/dL. Diabetes mellitus is suggested by the polyuria and the blood glucose of 380 mg/dL. Hypothyroidism is suggested by the elevated TSH. Patients with polyglandular deficiency syndrome, type II, may also present with transient hyperthyroidism secondary to destruction of follicles in the thyroid gland. Other features of the condition include antibodies directed against the target glands, particularly against cytochrome P450 adrenal cortical enzymes, and reduced systemic T- cell-mediated immunity. MEN type I (choice A) is characterized by tumors of the parathyroid glands, pancreatic islet cells, and pituitary gland. MEN type IIA (choice B) is characterized by medullary carcinoma of the thyroid, pheochromocytoma, and hyperparathyroidism. MEN type IIB (choice C) is characterized by multiple mucosal neuromas, marfanoid habitus, medullary carcinoma of the thyroid, and pheochromocytoma. Polyglandular deficiency syndrome type I (choice D) is characterized by onset in childhood or before age 35, hypoparathyroidism, adrenocortical failure, and gonadal failure; diabetes mellitus is not usually seen with this condition.
32
A 55-year-old white man is taken to the emergency department because he is vomiting fresh blood. For the past several hours, he had been vomiting, then dry heaving; 30 minutes ago, he suddenly started vomiting fresh blood. The patient’s temperature is 35.6 C (96 F), blood pressure is 110/55 mm Hg, pulse is 75/min, respirations are 16/min. Endoscopy demonstrates several lacerations in the mucosa near the gastroesophageal junction, Which of the following underlying conditions is likely present in this patient? (A) AIDS (B) Alcoholism (C) Chagas disease (D) Diabetes mellitus (E) Scleroderma
Respuesta: B The **correct answer is B**. These tears, called Mallory-Weiss lacerations, account for about 5% of cases of upper GI hemorrhage. They occur when the proximal part of the stomach is telescoped into the distal esophagus (stretching and tearing it) by severe vomiting, severe retching, or severe hiccups. The lacerations are usually superficial and often stop bleeding spontaneously. If the bleeding fails to stop spontaneously, the lacerations may be controlled endoscopically. The condition was initially described in alcoholics, but you should be aware that it occasionally occurs in many other types of patients as well. AIDS **(choice A)** can predispose for esophagitis due to viruses (CMV, herpes) or fungi (usually Candida). Chagas disease **(choice C)** can cause megaesophagus. Diabetes mellitus **(choice D)** can predispose for fungal (particularly Candida) esophagitis. Scleroderma **(choice E)** can involve the esophagus with fibrosis, leading to dysphagia.
33
A 34-year-old woman undergoes clinical investigations because of chronic dyspeptic symptoms, such as mild nausea, bloating after meals, and mild right upper abdominal pain. She is otherwise healthy and takes oral contraceptives only. Physical examination and laboratory studies are normal. Liver ultrasonography reveals a 5-cm, well-circumscribed mass in the right hepatic lobe. A percutaneous needle biopsy shows a lesion consistent with hepatic adenoma. When symptomatic, which of the following is the most common clinical presentation of this tumor? (A) Abdominal pain (B) Jaundice (C) Metastatic disease (D) Portal hypertension (E) Rupture into peritoneal cavity
Respuesta: E The **correct answer is E**. Liver adenoma is an infrequent benign tumor composed of hepatocytes arranged with the same lamellar pattern as the normal hepatic lobule. It is usually clinically silent and occurs most frequently in association with oral contraceptives or anabolic steroids. When symptomatic, its most common clinical presentation is rupture into the peritoneal cavity with consequent hemoperitoneum and shock. Abdominal pain **(choice A)** is a symptom of many hepatobiliary diseases, including hepatitis, carcinoma, and cholelithiasis. The pain of hepatic origin is presumed to result from stretching of the capsule as occurs in acute hepatitis. The pain of biliary origin derives from acute distention of the gallbladder, usually because of gallstones blocking the cystic or common bile duct. Jaundice **(choice B)** is not a manifestation of liver adenoma. Hemolytic disorders, diffuse hepatocellular damage, and obstruction of the biliary pathways are the most common conditions resulting in jaundice. Metastatic disease **(choice C)** is not a consequence of liver adenoma, which is a benign tumor. Liver adenoma, however, may be extremely similar histologically to well-differentiated hepatocellular carcinoma, so that one may be mistaken for the other. Portal hypertension **(choice D)** may result from prehepatic causes (thrombosis of the portal vein), hepatic causes (cirrhosis or other diffuse infiltrative conditions), and post-hepatic causes (obstruction of the hepatic veins or right-sided cardiac failure). Well-demarcated tumors, such as liver adenoma, do not lead to portal hypertension.
34
A 40-year-old man presents with the acute onset of severe pain in his left great toe for the past 24 hours. He denies trauma, fever, chills, or changes in appetite. He is a chronic smoker but does not abuse alcohol. He is currently on day 4 of a course of erythromycin for bronchitis. Examination shows his blood pressure is 120/80 mm Hg, pulse is 80/min, and respirations are 18/min. The left great toe is red, swollen, and tender to touch, with no fluctuation. Laboratory evaluation shows the following: Sodium 139 mEq/L Potassium 4.5 mEq/L Hemoglobin 13.5 g/dL Leukocytes 9000/mm3 Platelets 350,000/mm3 Uric acid 14 mg/dL Which of the following is the most appropriate initial step in management? (A) Allopurinol (B) Colchicine (C) Indomethacin (D) Intra-articular corticosteroids (E) Probenecid
Respuesta: C The **correct answer is C**. The classic presentation, along with the hyperuricemia, makes gouty arthritis the most likely diagnosis. Nonsteroidal anti-inflammatory drugs (NSAIDs) are the treatment of choice of acute gouty arthritis. They are somewhat less specific than colchicine for gout but are better tolerated and work quickly. Allopurinol **(choice A)** is used in the intercritical period between attacks, not in acute attacks. Colchicine **(choice B)** is effective for acute gouty arthritis but is poorly tolerated at the high, frequent doses required for an acute attack. Steroids **(choice D)** can be used in an acute episode, but not before a trial with NSAIDs and/or colchicine is attempted. Probenecid **(choice E)** is a uricosuric agent used in the intercritical period, not during acute attacks, in patients who are under excreters of uric acid.
35
A 38-year-old engineer presents with complaints of epistaxis, severe sinus pain, and purulent sinus drainage over the past 6 months. One year ago, he had frequent episodes of cough, and a chest x-ray film obtained at that time showed pulmonary infiltrates. He recalls that his cough was poorly responsive to antibiotics. The man also admits to general weakness and diffuse muscle aches. Over the past 8 months he has lost more than 5 kg (10 lb). The patient has no other medical problems, denies smoking, is not taking any medications, and reports no other symptoms. On physical examination, his lungs are clear without wheezing or rhonchi. Small ulcerative lesions are apparent on his nasal septum, and many tender subcutaneous nodules are present on his back and arms. Laboratory values are as follows: Erythrocyte sedimentation rate 66 mm/h Leukocytes 12,000/mm3 (70% segmented neutrophils, 20% lymphocytes, 10% monocytes) Hemoglobin 13 g/dL Hematocrit 41% A chest x-ray film is obtained that shows multiple 1- to 2-cm nodules in both upper lung fields. Which of the following is most likely to confirm the diagnosis? (A) Antiribonucleoprotein (B) Elevated serum IgE (C) Positive antiglomerular basement membrane antibody (D) Positive c-ANCA (E) Positive p-ANCA (F) Positive rheumatoid factor
Respuesta: D The **correct answer is D**. Wegener granulomatosis is a vasculitis that affects the upper respiratory tract and paranasal sinuses, the kidneys, and the lungs. It is associated with dermatologic symptoms (subcutaneous nodules and purple papules). Antineutrophil cytoplasmic antibodies yielding a cytoplasmic immunofluorescence pattern (c-ANCA) are found in more than 90% of patients with Wegener granulomatosis. Antiribonucleoprotein **(choice A)** is found in high titer levels in mixed connective tissue disease (MCTD). MCTD is a distinct rheumatic syndrome characterized by overlapping clinical features of systemic lupus erythematosus, scleroderma, polymyositis or dermatomyositis, and rheumatoid arthritis. Elevated levels of serum IgE **(choice B)** might be seen in acute bronchopulmonary aspergillosis. Goodpasture syndrome is associated with anti-glomerular basement membrane antibody **(choice C)**. This disorder affects the kidney and lungs, rather than the lungs or paranasal sinuses. A positive p-ANCA **(choice E)** is seen in polyarteritis nodosa, a nongranulomatous vasculitis. Purple nodules can be seen in both polyarteritis nodosa and Wegener granulomatosis. Rheumatoid factor is positive **(choice F)** in many patients with rheumatoid arthritis, but this marker is nonspecific and may be positive in several other disorders.
36
An 87-year-old woman with advanced Alzheimer disease is transferred from a nursing home to a hospital unit after a deterioration in her condition. Physical examination demonstrates a nearly immobile, emaciated woman with poor skin turgor and a dry mouth. Her temperature is 35.6 C (96 F), blood pressure is 90/60 mm Hg, pulse is 90/min, and respirations are 15/min. Blood chemistry studies demonstrate a sodium of 151 mEq/L, a potassium of 5.3 mEq/L, and a chloride of 112 mEq/L. Which of the following is the most appropriate next step in management? (A) Oral water administration (B) IV 0.9% saline (C) IV 5% D/W (D) IV colloid and 0.9% saline (E) IV distilled water
Respuesta: C The **correct answer is C**. This is a dehydrated and malnourished patient with hypernatremia. The most probable cause is that she is no longer able to express thirst or hunger and cannot cooperate efficiently with caregivers when taking water or food. The most immediate goal of therapy is to replace water. The usual choice in this setting is 5% dextrose in water, which should be given relatively slowly to prevent glucosuria, followed by normal saline. This strategy would increase the salt-free water excretion and hypertonicity. Oral water administration **(choice A)** will work with physically and mentally competent people who have been water deprived (e.g., stranded in the desert). However, this strategy has obviously already failed in this patient (the nursing home was probably not deliberately trying to dehydrate her) and is not the best choice. IV 0.9% saline **(choice B)** by itself would not be used in this clinical setting. IV colloid and 0.9% saline **(choice D)** is used as the initial hydrating fluid in patients who are in shock secondary to dehydration, but is not needed in this patient with adequate blood pressure and symptomatic hypernatremia. IV distilled water **(choice E)** should never be given to patients-it will cause red cell lysis.
37
A 28-year-old man presents to a physician because of sores on his penis and scrotum. He also has lesions in his oral cavity, which began a few days before the sores on his genitalia. He has had several similar episodes in the past, but they resolved before he consulted a physician. During these periods, his eyes often hurt when he goes into bright light. He has also had recent intermittent pain in his knees. Physical examination demonstrates oral lesions resembling aphthous ulcers, as well as small ulcers on the genitalia. Tzanck smear of the genital and oral ulcers is negative for multinucleated giant cells. Blood studies demonstrate an elevated erythrocyte sedimentation rate (ESR), elevated alpha-2 and gamma globulins, and mild leukocytosis. Which of the following is the most likely diagnosis? (A) Behcet syndrome (B) Herpes simplex infection (C) Neisseria gonorrhoeae infection (D) Psoriasis (E) Treponema pallidum infection
Respuesta: A The **correct answer is A**. This is Behcet syndrome. The laboratory findings are usually nonspecific indications of an inflammatory process (such as those illustrated in the question stem), so the diagnosis is usually established by the history (and may take months to years since not all features are typically present from the beginning). Characteristic features include painful oral ulcers, painful genital ulcers (in men, in women they may be painless), ocular disease (most often a relapsing iridocyclitis that causes pain and photophobia), skin lesions (papules, pustules, vesicles, or folliculitis), and mild arthritis of large joints. Other features that may be seen include CNS involvement and migratory thrombophlebitis. Herpes simplex infection **(choice B)** can cause oral and genital ulcers, but the Tzanck smear would probably be positive and arthritis would be unlikely. Neisseria gonorrhoeae infection **(choice C)** can affect both genitalia and joints, but does not usually cause ulcers. Psoriasis **(choice D)** can cause both skin lesions and arthritis, but the skin lesions are characteristically scale-covered plaques. Treponema pallidum infection **(choice E)** can cause ulcers (the chancre) but does not usually cause arthritis.
38
A 65-year-old man reports feeling increasingly tired and short of breath. He had been well until a year ago, when he started losing weight despite any dietary change. He also complains of pencil thin stools. His wife has commented that he seems very pale. He is on no medications. Physical examination shows a pale-appearing man with an elevated heart rate. Rectal examination is positive for occult blood. Laboratory studies are remarkable for a hematocrit of 25%. Which of the following is the most appropriate next step in diagnosis? (A) KUB (x-ray film showing the kidney, ureters and bladder) (B) Colonoscopy (C) Esophagoduodenoscopy (D) Sigmoidoscopy (E) Open laparotomy
Respuesta: B The **correct answer is B**. This patient may have colon cancer. The weight loss, pallor, and anemia are indicative of a chronic bleed, supported by the stool positivity for occult blood. Risk factors for colon cancer include a positive history of colon cancer or adenomatous polyps in a first-degree relative and a personal history of adenomatous polyps. Since our suspicion of colorectal cancer is high, it would be helpful to detect the lesion and remove it if possible. Colonoscopy provides the most efficient modality with which to achieve this goal. A KUB **(choice A)** is usually a good modality for evaluating obstruction, which is a potential complication of colon cancer. This patient is not complaining of obstruction, and a KUB would not detect a soft tissue mass. Esophagoduodenoscopy **(choice C)** would be done to evaluate the patient for a source of upper gastrointestinal bleeding, such as ulcers, varices, or gastric cancer. Sigmoidoscopy **(choice D)** would be an effective screening test, but only 25 to 30% of colorectal cancers are detected by rigid sigmoidoscopy; the rate increases to 40 to 65% when flexible sigmoidoscopy is used. This patient has a high pre-test probability of having colorectal cancer, and since visualization of the entire colon will guide further action, sigmoidoscopy would be inadequate. Open laparotomy **(choice E)** might ultimately be needed if the patient undergoes colonic resection; however, it would be premature to immediately proceed to this option.
39
A 42-year-old woman with a history of rheumatoid arthritis develops tingling, pain, and focal numbness of one hand. These sensations involve the palmar aspect of the thumb and fingers, with the exception of the ulnar edge of the little finger. The woman works as a secretary and has noticed that her symptoms are worst at night, particularly after she has done considerable typing during the preceding workday. Tapping on the volar surface of the wrist with a reflex hammer reproduces the symptoms. Which of the following is the most likely diagnosis? (A) Carpal tunnel syndrome (B) Cubital tunnel syndrome (C) Radial tunnel syndrome (D) Reflex sympathetic dystrophy (E) Scapholunate ligament rupture
Respuesta: A The **correct answer is A**. The patient probably has carpal tunnel syndrome, which is caused by compression of the median nerve as it passes through the carpal tunnel in the wrist. The symptoms illustrated in the question stem are typical, as is the history of exacerbation of symptoms at night following heavy wrist use during the day. The test described is the Tinel test, which can be used to assess other superficial nerves as well. Milder cases of carpal tunnel syndrome may respond to rest and nonsteroidal anti-inflammatory drugs; more severe cases may require surgical decompression. Cubital tunnel syndrome **(choice B)** compresses the ulnar nerve at the elbow, producing numbness and paresthesias of the ring and little fingers. Radial tunnel syndrome **(choice C)** involves compression of branches of the radial nerve in the arm or forearm. It causes pain of the back of the forearm and hand, sometimes with wrist drop. Reflex sympathetic dystrophy **(choice D)** is pain and limited motion of the shoulder accompanied by ipsilateral involvement of the hand. Scapholunate ligament rupture **(choice E)** usually occurs during a fall onto an outstretched hand and causes pain in the mid-wrist.
40
An 18-year-old student comes to the physician 5 hours after sustaining an injury to his right ankle while playing football. He says that he “rolled his ankle over,” but denies feeling any snap or crack at the time of injury. He walks with a slight limp. Examination shows swelling of the right ankle, especially on the lateral side. Pulses and sensation are normal. Pain is most pronounced in the area of the calcaneofibular ligament. The “drawer” test is normal, but the “talar tilt” test reveals increased excursion on the right compared with the uninjured side. There is no tenderness on palpation of the lateral malleolus. Which of the following is the most appropriate next step in management? (A) Referral to orthopedic specialist (B) Treatment with nonsteroidal anti-inflammatory drugs without ankle immobilization (C) Treatment for acute ankle sprain with rehabilitation within 72 hours (D) Treatment for acute ankle sprain with rehabilitation after 10 days of ankle rest (E) X-ray examination
Respuesta: C The **correct answer is C**. This patient presents with a lateral ankle sprain. Clinical assessment is usually sufficient to diagnose this condition, unless there are signs suggesting the presence of fracture. The drawer and talar tilt tests are used to assess ligament instability and confirm a diagnosis. A positive talar tilt test indicates injury to the calcaneofibular ligament, and a positive drawer test indicates injury to the anterior talofibular ligament. The history is also very important in clarifying the mode of traumatic injury. Currently recommended treatment for uncomplicated ankle sprains of mild-to-moderate degree includes protection of the injured joint by splinting or immobilizing boot; rest of the injured joint; ice application several times daily; compression by elastic wrap, and elevation of the limb to reduce edema (which gives the PRICE mnemonic). Adjuvant anti-inflammatory or analgesic treatment may be used. Patients with lateral ankle sprains should start rehabilitation early, usually after 24-72 hours of ankle rest. Treatment for acute ankle sprain with rehabilitation after 10 days of ankle rest **(choice D)** has been found to be unnecessary compared with shorter periods of immobilization. Referral to orthopedic specialist **(choice A)** is rarely necessary, unless there are complicating factors, such as associated injuries to other joints or fractures. Treatment with nonsteroidal anti-inflammatory drugs without ankle immobilization **(choice B)** is definitely inappropriate in any case of ankle sprain, however mild. X-ray examination **(choice E)** is indicated when there are signs suggestive of fractures, such as history of a snapping or popping sound at the time of injury, pain in the malleolar region, and tenderness on pressure on the lateral malleolus. It is recommended that patients older than 55 years should undergo routine x-ray control even in the absence of clinical signs of fractures because of the increased incidence of occult fractures in more advanced age.
41
One week following an uncomplicated delivery, a 27-year-old woman comes to the physician because of polyuria and excessive thirst. Her pregnancy was normal, but she reports that she began to feel an unusual craving for ice water in the last month of gestation. She is forced to get up at night several times to void and drinks large amounts of water. Her blood pressure is 120/80 mm Hg. Serum electrolytes are within normal limits. A 24-hour urine collection during ad libitum water intake yields the following results: Total volume 10 L Specific gravity < 1.006 Glucose Absent Protein < 150 mg Which of the following is the most appropriate next step in management? (A) Advise the patient to reduce water intake (B) Perform vasopressin challenge test (C) Order MRI studies of the pituitary-hypothalamic region (D) Refer for psychiatric evaluation of compulsive water drinking (E) Refer to nephrologist for evaluation of nephrogenic diabetes insipidus
Respuesta: B The **correct answer is B**. The clinical picture is highly suggestive of diabetes insipidus, a disorder due to deficiency of antidiuretic hormone (ADH) secretion from the posterior pituitary gland. Polydipsia is a consequence of polyuria (not an effect) because of abundant diuresis secondary to ADH deficiency. The vasopressin challenge test is given by administering desmopressin (a synthetic form) by the nasal route, and monitoring the urine output 12 hours before and 12 hours after administration. If the symptomatology is due to central diabetes insipidus, the patient will experience an immediate reduction in thirst and urine output. Diabetes insipidus may manifest in the third trimester of pregnancy or during the puerperium because of a circulating enzyme (vasopressinase) that degrades vasopressin. The enzyme is not effective against desmopressin. The disorder resolves spontaneously. Besides the high likelihood of noncompliance, advising the patient to reduce water intake **(choice A)** would expose her to severe dehydration and hypernatremia due to loss of water and resultant hemoconcentration. MRI studies of the pituitary-hypothalamic region **(choice C)** are performed to search for mass lesions that may be the underlying cause of central diabetes insipidus. These investigations should be performed after the diagnosis has received confirmation by a vasopressin challenge test. Compulsive water drinking may require psychiatric evaluation **(choice D)**, and may mimic diabetes insipidus, but this possibility should be undertaken once diabetes insipidus has been ruled out. In this particular case, the close temporal association with pregnancy and delivery makes the diagnosis of diabetes insipidus more likely. Evaluation of nephrogenic diabetes insipidus **(choice E)** is the next appropriate step if the patient does not respond to vasopressin challenge test. Nephrogenic diabetes insipidus, in fact, is due to resistance of renal tubules to the action of vasopressin. A hereditary X-linked form is known, usually associated with hyperuricemia. Acquired forms are associated with a variety of conditions, e.g., pyelonephritis, multiple myeloma, and chronic hypercalcemia.
42
A 74-year-old man is hospitalized for treatment of enterococcal pneumonia. He has a history of gout and hyperuricemia, for which he takes allopurinol. In the hospital, the patient receives treatment with ampicillin and gentamicin for 7 days. Despite antibiotic therapy, he remains persistently febrile. On the 8th hospital day, he manifests signs of rapidly progressive renal insufficiency. His temperature is 38.2 C (100.8 F), blood pressure is 100/60 mm Hg, pulse is 100/min, and respirations are 20/min. Laboratory investigations show: - Blood exams: Sodium 145 mEq/L Potassium 6.5 mEq/L Chloride 110 mEq/L Bicarbonate 20 mEq/L Urea nitrogen 40 mg/dL Creatinine 3.5 mg/dL Uric acid 8.5 mg/dL - Urine: Sodium 23 mEq/L Creatinine 35 mg/dL Pigmented granular casts Present Protein Negative Erythrocytes Negative Which of the following is the most likely cause of this patient’s renal failure? (A) Allopurinol toxicity (B) Ampicillin toxicity (C) Gentamicin toxicity (D) Hyperuricemia (E) Hypotension (F) Sepsis
Respuesta: C The **correct answer is C**. Acute renal failure (ARF) is a common occurrence in hospitalized patients. It is crucial to determine whether ARF is secondary to prerenal or renal causes. In the former case, the underlying cause is reduction of blood flow to normal kidneys, resulting in a decreased glomerular filtration rate. In the latter, intrinsic renal damage is the underlying etiology. Of the intrinsic causes of ARF, acute tubular necrosis is the most common. How can one distinguish between prerenal and renal forms of ARF? Sodium reabsorption is not impaired in prerenal azotemia, whereas creatinine reabsorption is deficient in both prerenal and intrinsic renal azotemia. Fractional excretion of sodium (FENa) is therefore a most useful parameter to distinguish between these two conditions. It can be calculated by the following formula: NaURINE × CrPLASMA/CrURINE × NaPLASMA, × 100. In prerenal azotemia, FENa < 1% because the undamaged renal tubules will avidly absorb sodium. In acute tubular necrosis, as well as other intrinsic renal causes of azotemia, damaged tubules will allow sodium to leak into urine, and FENa will be >1%. In this particular case, the calculation yields FENa = 1.6%. Gentamicin is the most nephrotoxic of the aminoglycoside antibiotics. Its toxicity usually manifests after 5-7 days of treatment. The most frequent manifestation of allopurinol toxicity **(choice A)** is a pruritic rash due to hypersensitivity. Vasculitis and hepatitis are other, albeit rare, adverse effects. Ampicillin toxicity **(choice B)** may manifest with renal damage, but usually in the form of acute interstitial nephritis, not acute tubular necrosis. Acute interstitial nephritis is associated with fever, rash, blood eosinophilia, and leukocyturia with eosinophils. Hyperuricemia **(choice D)** may cause acute tubular necrosis, but only when serum uric acid levels rise rapidly as a result of rapid cell turnover. This may develop with hematologic malignancies or germ cell neoplasms treated with chemotherapy. Uric acid levels are often very high (>20 mg/dL). Hypotension **(choice E)** is the underlying pathogenetic factor of most conditions leading to prerenal azotemia, such as hypovolemic, cardiogenic, or anaphylactic shock. Renal tubular function is preserved, but the fall in glomerular filtration rate results in oliguria/anuria and ARF. FENa is very low (< 1%). Clinical history is obviously an important aid in the differential diagnosis between prerenal and renal azotemia. Sepsis **(choice F)** is a frequent cause of prerenal azotemia when associated with shock. In this case, the laboratory data rule out this possibility.
43
A 72-year-old woman is driven to the emergency department because 10 minutes earlier she had developed painless, sudden, unilateral blindness. When light is shined on the affected eye, the pupil fails to constrict. When the light is shined on the opposite eye, the affected eye’s pupil constricts briskly. Tonometry of both eyes is within normal limits. Ophthalmoscopy demonstrates a pale, opaque fundus with a red fovea. The arteries are markedly attenuated. Which of the following is the most likely diagnosis? (A) Age-related macular degeneration (A) Age-related macular degeneration (B) Central retinal artery occlusion(B) Central retinal artery occlusion (C) Central retinal vein occlusion (D) Hypertensive retinopathy (E) Retinitis pigmentosa
Respuesta: B The **correct answer is B**. The presentation is classic for central retinal artery occlusion, which is a blockage of the central retinal artery by embolism or thrombosis that causes painless, sudden, unilateral blindness. Patients may have underlying atherosclerosis, endocarditis, or temporal arteritis. The retinal changes illustrated in the question stem are typical and are the result of a failure of blood to flow into the retina. Immediate treatment is imperative. Intermittent digital massage over the closed eyelids may dislodge the embolus and allow it to flow into a smaller blood vessel, where it will cause a small area of retinal ischemia. If this fails, anterior chamber paracentesis will also sometimes dislodge the embolus. Age-related macular degeneration **(choice A)** can also present with sudden, painless, unilateral blindness, but the fundus will show pigmentary changes with or without new vessel formation behind the retina. Central retinal vein occlusion **(choice C)** can also cause painless, unilateral blindness. However, it tends to develop a little more slowly than central artery occlusion, and the retinal vessels appear congested. Hypertensive retinopathy **(choice D)** does not usually cause sudden blindness. Retinitis pigmentosa **(choice E)** develops over a period of years, with loss of peripheral vision.
44
A 45-year-old woman presents to a physician with marked swelling of her hands. Five years previously, she had developed multiple arthralgias, and early rheumatoid arthritis was suspected; however, no confirmatory immunologic studies had been performed at that time. Since then, she has experienced difficulty swallowing and mild dyspnea on exertion. On physical examination, the hands are strikingly swollen, producing a sausagelike appearance to the fingers. Other findings include erythematous patches over the knuckles, a mild malar rash, and slight violaceous discoloration to the eyelids. No joint deformity is noted, although many joints are tender. A chest x-ray film demonstrates diffuse interstitial infiltrates. In antibody studies, high titers of antibodies directed against which of the following antigens will most likely be present? (A) c-ANCA (B) dsDNA (C) p-ANCA (D) Scl-70 (E) RNP
Respuesta: E The **correct answer is E**. This patient has mixed connective tissue disease. Clinically, it appears to be an overlap syndrome with features similar to rheumatoid arthritis, systemic lupus erythematosus, scleroderma, Sjögren syndrome, and polymyositis or dermatomyositis. It is now considered a separate disease because of a distinct autoantibody pattern, with very high levels of antibody directed against ribonucleoprotein. Rheumatoid agglutinins may also be present in high titers. Other antibodies characteristic of the individual diseases that mixed connective tissue disease mimics are usually absent or present only in low titers. The clinical presentation may be wildly diverse, depending on which disease pattern manifests first. With time, however, there should be clinical features suggestive of a variety of different autoimmune diseases. Mild disease may be controlled with measures similar to mild rheumatoid arthritis (salicylates, other NSAIDs, antimalarials, very-low-dose corticosteroids); more severe disease (which may be fatal with complications due to vascular lesions, renal failure, myocardial infarction, disseminated infection, or cerebral hemorrhage) usually requires large-dose steroids. Associate c-ANCA **(choice A)** with Wegener granulomatosis. Associate dsDNA **(choice B)** with systemic lupus erythematosus. Associate p-ANCA **(choice C)** with microscopic polyarteritis. Associate Scl-70 **(choice D)** with systemic sclerosis.
45
A 41-year-old high school teacher comes to the physician for a health maintenance examination. He has been in good health except for occasional episodes of acute upper respiratory infections. His blood pressure is 137/83 mm Hg. His height is 175 cm (69 in), and his weight is 81 kg (180 lb). He has no family history of early coronary artery disease, hypertension, or hyperlipidemia. He drinks alcohol on social occasions and has been smoking 30 cigarettes daily since the age of 20. He smokes his first cigarette soon after getting up in the morning, finds it extremely difficult to refrain from smoking while teaching in school, and does not quit even when he is sick in bed. With respect to smoking cessation, which of the following is the most appropriate next step in management? (A) Ask patient if he intends to quit smoking in the near future (B) Try to elicit a quitting date even if patient appears unwilling (C) Prescribe nicotine patch and gum (D) Recommend group behavior treatment (E) Refer to a psychiatrist for alcohol and nicotine dependence
Respuesta: A The **correct answer is A**. Cigarette smoking is considered the principal preventable cause of disease in the U.S., and approximately 20% of adults in this country smoke. The guidelines of the U.S. Agency for Health Care Policy and Research (AHCPR) and the National Cancer Institute recommend that all primary care physicians identify smokers and advise them to quit smoking. Several studies have found that 10% of all smokers will quit smoking if they receive even a 3-minute advice to quit from a clinician. However, the very first step in the physician’s approach to a smoker is to establish his/her willingness to quit. With respect to smoking cessation, three stages have been identified: precontemplation, if there is no intention of quitting; contemplation, if there are some ideas but no clear plans of quitting soon; and action, if there is a definite intention of setting a quitting date. The next steps depend on the patient’s response. Trying to elicit a quitting date even if the patient appears unwilling **(choice B)** would be inappropriate. The physician should elicit a quitting date if the patient is already willing to do so, i.e., he/she is in the contemplation or action stage. The physician should discuss the reasons or obstacles that make a patient unwilling to stop if he/she is in the precontemplation stage. Information about risks of smoking should be provided. Prescribing nicotine patch and gum **(choice C)** is most useful in smokers who are nicotine dependent. Several questionnaires have been developed to establish nicotine dependence (the Fagerstrom questionnaire is probably the most widely used). Nicotine dependence is present if a patient smokes the first cigarette within 30 minutes after awakening, smokes more than 20 cigarettes daily, finds it difficult to refrain from smoking in places where smoking is prohibited, and continues to smoke even during illness. Determining nicotine dependence allows a physician to individualize treatment strategies by prescribing nicotine replacement therapy. Group behavior treatment **(choice D)** is mostly indicated for smokers who have already tried and failed to quit in the past. Recent studies have shown that combinations of intensive group counseling with nicotine replacement have the highest success rate (up to 40%) in obtaining long-term abstinence. Maximal specialized care with intervention of a psychiatry specialist **(choice E)** may be indicated for patients with a high degree of nicotine dependence associated with other forms of drug abuse (alcohol being the most frequent).
46
A 21-year-old woman presents to the emergency department with several days of fever and fatigue. Her temperature is 39.0 C (102.2 F), her pulse is 90/min, and her blood pressure is 96/64 mm Hg. Her chest is clear to auscultation, and her abdomen is soft and nontender. A grade III/IV holosystolic murmur that increases on inspiration is best heard along the left sternal border. A CBC shows normochromic normocytic anemia. Chest x-ray films reveal several well-circumscribed, round infiltrates in multiple lobes. Echocardiography and blood cultures suggest a diagnosis of acute bacterial endocarditis limited to the tricuspid valve. Which of the following is the most likely explanation for these findings? (A) Congenital heart disease (B) Illicit drug use (C) Rheumatic fever (D) Rheumatoid arthritis (E) Systemic lupus erythematosus
Respuesta: B The **correct answer is B**. The most probable etiology of bacterial endocarditis involving the tricuspid valve is illicit IV drug use, which can introduce skin organisms into the venous system that then proceed to attack the tricuspid valve. Staphylococcus aureus accounts for between 60 and 90% of cases of endocarditis in IV drug users. The endocarditis associated with congenital heart disease **(choice A)** typically involves either damaged valves or atrial or ventricular septal defects. The tricuspid valve is not particularly vulnerable. Rheumatic fever **(choice C)** most commonly damages the mitral and aortic valves, and tricuspid damage is usually less severe and seen only when the mitral and aortic valves are heavily involved. Consequently, secondary bacterial endocarditis involving only the tricuspid valve in a patient with a history of rheumatic fever would be unusual. Rheumatoid arthritis **(choice D)** is not associated with bacterial endocarditis. Systemic lupus erythematosus **(choice E)** can produce small, aseptic vegetations on valves (Libman-Sacks endocarditis) but is not associated with bacterial endocarditis.
47
A 22-year-old man is brought to the emergency department several hours after sustaining a severe head trauma during a soccer game. He became unconscious for a short time soon after the accident, regained consciousness for 3 hours, and subsequently relapsed into coma. On arrival, the patient appears unresponsive to verbal or painful stimuli. A scalp lesion consistent with prior contusion is found in the right parietal region. The right pupil is dilated and poorly reactive to light, and funduscopic examination reveals early papilledema. X-ray films of the head show a linear fracture in the right calvarial wall. Which of the following is the most likely diagnosis? (A) Epidural hemorrhage (B) Fracture without associated brain injury (C) Intracerebral hypertensive hemorrhage (D) Subarachnoid hemorrhage (E) Subdural hemorrhage
Respuesta: A The **correct answer is A**. The clinical presentation is highly characteristic of epidural bleeding, which usually is of traumatic origin and most often results from rupture of the middle meningeal artery. The initial concussion leads to a brief loss of consciousness, which is followed by a lucid interval lasting several hours. As the epidural hematoma progressively enlarges and pushes the underlying brain, the patient becomes comatose again and may display signs of uncal herniation. The herniating uncus pushes on the third cranial nerve, producing ipsilateral fixed pupillary dilatation. Papilledema is usually a late sign and indicates cerebral edema. A calvarial fracture **(choice B)** would not cause such a severe neurologic state and evidence of uncal herniation unless associated with an intracranial hematoma. Intracerebral hypertensive hemorrhage **(choice C)** usually occurs spontaneously and develops within the brain parenchyma (most commonly in the basal ganglia). Loss of consciousness develops in approximately 50% of patients. Headache, vomiting, and variable neurologic deficits are present. Subarachnoid hemorrhage **(choice D)** characteristically manifests with sudden onset (thunderclap) of headache associated with vomiting and progressive impairment of consciousness. The most frequent cause is rupture of berry aneurysms. The clinical manifestations of a subdural hemorrhage **(choice E)** may vary depending on the severity and location. Cerebral atrophy is a predisposing condition, as it leads to “stretching” of bridging veins that connect the veins on the cerebral convexities with the superior sagittal sinus. Minimal trauma may then result in tearing of such veins. Impaired consciousness and/or focal neurologic deficits follow the traumatic event after an interval of days or weeks.
48
A 72-year-old man suffers severe pleuritic chest pain on the seventh postoperative day after surgical pinning of an intertrochanteric hip fracture. He was not on anticoagulants at the time. He is hemodynamically stable, but is short of breath and has distended neck veins. Spiral CT scan shows the presence of a pulmonary embolus on the left side, and echocardiogram does not show signs of right ventricular strain. Which of the following is the most appropriate management at this time? (A) Anticoagulation with heparin (B) Infusion of thrombolytic agents into the left pulmonary artery (C) Insertion of vena cava filter (D) Surgical embolectomy (E) Systemic infusion of thrombolytic agents
Respuesta: A The **correct answer is A**. Anticoagulation with heparin is the standard therapeutic first step for pulmonary embolus. Thrombolytic agents **(choices B and E)** still have a very limited role in the treatment of pulmonary embolus. They may be considered when overwhelming right-sided heart failure is life threatening, but bleeding remains a formidable potential complication; therefore, they are contraindicated after major surgery. When they are used, systemic administration **(choice E)** is as effective as direct delivery into the pulmonary artery **(choice B)**. Vena cava filters **(choice C)** are indicated only when pulmonary emboli recur while under anticoagulation, or if anticoagulation cannot be instituted. Surgical embolectomy **(choice D)** has the same restricted indications noted for the thrombolytic agents, and as a rule it is a major undertaking rarely feasible in a very sick patient.
49
A 60-year-old man presents to the physician with his daughter, who is concerned about her father’s insidious onset of inattention and slowing of thought processes. The man reports difficulty walking and a recent onset of urine incontinence. On physical examination, his gait is noted to be unsteady, slow, and wide-based with a shuffling quality. His past medical history and family history are noncontributory. MRI of the brain is performed and reveals dilated ventricular spaces with a relatively preserved cortical mantle. A shunting procedure is performed and leads to an improvement of his clinical condition. (A) Alzheimer disease (B) Chronic subdural hematoma (C) Creutzfeldt-Jakob disease (D) Dementia pugilistica (E) Depression (F) Diffuse Lewy body disease (G) Glioblastoma multiforme (H) HIV encephalopathy (I) Meningioma (J) Normal pressure hydrocephalus (K) Parkinson disease (L) Pellagra (M) Pick dementia (N) Vascular dementia (O) Wernicke encephalopathy/Korsakoff syndrome
Respuesta: J The **correct answer is J**. The triad of dementia, gait disturbances, and incontinence should immediately suggest normal pressure hydrocephalus as the underlying etiology (although such a complete clinical picture is often not present). MRI usually reveals an obvious discrepancy between the marked degree of ventricular dilatation (hydrocephalus) and the comparatively lesser extent of cortical atrophy. This cause of dementia is important because it is potentially treatable with a shunting procedure. However, long-standing cases lead to cortical atrophy and consequently become less responsive to shunting.
50
A 60-year-old woman is referred to a neurologist because of progressive loss of memory, disturbance of language, and social withdrawal. On examination, she is noted to have poor personal hygiene, and she acts in a disinhibited manner. Mental status examination reveals difficulty in naming objects, echolalia, poor insight, and judgment. There is a mild disturbance of memory, but visual-spatial functions are preserved. An MRI of the brain is performed and reveals marked cortical atrophy of the frontal lobes and anterior temporal lobes, with relative sparing of the remaining cortex. (A) Alzheimer disease (B) Chronic subdural hematoma (C) Creutzfeldt-Jakob disease (D) Dementia pugilistica (E) Depression (F) Diffuse Lewy body disease (G) Glioblastoma multiforme (H) HIV encephalopathy (I) Meningioma (J) Normal pressure hydrocephalus (K) Parkinson disease (L) Pellagra (M) Pick dementia (N) Vascular dementia (O) Wernicke encephalopathy/Korsakoff syndrome
Respuesta: M The **correct answer is M**. The clinical picture of Pick dementia may be confused with other forms of dementia, especially Alzheimer disease, although in its classic presentation, flat emotional affect and language disturbances are predominant. However, MRI evidence of marked atrophy limited to the frontal and anterior temporal cortex supports a diagnosis of Pick disease. If a cortical biopsy is performed, it will show neuron loss with other characteristic neuronal changes, namely intracytoplasmic argyrophilic inclusions. Alzheimer disease **(choice A)** is the most frequent form of dementia and certainly should be suspected in all cases of gradually progressive memory loss accompanied by MRI evidence of diffuse cerebral atrophy. Dilatation of ventricles is secondary to loss of cerebral substance and is referred to as hydrocephalus ex vacuo. Chronic subdural hematoma **(choice B)** refers to a subdural collection of blood resulting from tearing of bridging veins. This may slowly enlarge and compress the underlying brain, occasionally resulting in dementia. The diagnosis is easily made by CT/MRI studies. Creutzfeldt-Jakob disease **(choice C)** causes a rapidly progressive form of dementia, accompanied by abnormal myoclonic movements and characteristic EEG changes. MRI does not reveal significant cerebral atrophy. Dementia pugilistica **(choice D)** is associated with a history of boxing. Cerebral atrophy is evident on MRI. Depression **(choice E)** is a major cause of pseudodementia, when intellectual impairment is not due to neurodegenerative changes. Other manifestations of depression are usually evident. Diffuse Lewy body disease **(choice F)** is now ranked among the most common causes of dementia. Neuron loss and Lewy bodies (the same as in Parkinson disease) are found in the substantia nigra and in several neocortical fields, leading to parkinsonism, dementia, and other neurologic manifestations such as visual hallucinations. Dementia is a very unusual presentation of intracranial tumors such as glioblastoma multiforme **(choice G)** and meningioma **(choice I)**. Glioblastoma multiforme is the most frequent primary malignant tumor, and meningioma is the most frequent benign tumor of the brain. Both can be detected by MRI. HIV encephalopathy **(choice H)** is referred to as AIDS-dementia complex or HIV-associated cognitive/motor complex. It manifests in AIDS patients with progressive deterioration of cognitive function and slowing of motor tasks (e.g., handwriting). MRI and CSF studies are necessary to exclude opportunistic infections or lymphoma. One quarter of Parkinson disease patients **(choice K)** presents with dementia. Three cardinal signs of Parkinson disease are resting tremor, rigidity, and bradykinesia, two of which are required for clinical diagnosis. Postural instability is the fourth cardinal sign and emerges late, usually after 8 years. Dementia also occurs late in the disease and affects short-term memory and visual spatial function. Pellagra **(choice L)** is due to nicotinic acid deficiency, which in its advanced stages manifests with the classic triad of dermatitis, diarrhea, and dementia. Vascular dementia **(choice N)** refers to dementia resulting from vascular causes, the most frequent of which are atherosclerosis of major cerebral arteries and hypertensive arteriolosclerosis. MRI would reveal multiple old infarcts in the cerebral cortex or lacunar infarcts in the basal ganglia, combined with variable white matter rarefaction. Wernicke encephalopathy and Korsakoff syndrome **(choice O)** are both seen in chronic alcoholics and are due to thiamine deficiency. The former consists of nystagmus, ataxia, and confusion; the latter consists of impaired memory with confabulation.